Вы находитесь на странице: 1из 165

ENGINEERING MATHEMATICS-I 15MAT11

SYLLABUS
Engineering Mathematics-I
Subject Code: 15MAT11 IA Marks: 20
Hours/Week: 04 Exam. Hours: 03
Total Hours: 50 Exam. Marks: 80

Course Objectives
To enable students to apply knowledge of Mathematics in various
engineering fields by making hem to learn the following:
• nth derivatives of product of two functions and polar curves.
• Partial derivatives.
• Vectors calculus.
• Reduction formulae of integration to solve First order
differential equations
• Solution of system of equations and quadratic forms.

Module –1
Differential Calculus -1:
Determination of nth order derivatives of Standard functions -
Problems. Leibnitz‟s theorem (without proof) - problems.
Polar Curves - angle between the radius vector and tangent, angle
between two curves, Pedal equation for polar curves. Derivative of
arc length - Cartesian, Parametric and Polar forms (without proof)
- problems. Curvature and Radius of Curvature – Cartesian,
Parametric, Polar and Pedal forms(without proof) and problems.
10hrs

Module –2
Differential Calculus -2
Taylor‟s and Maclaurin‟s theorems for function of o ne
variable(statement only)- problems. Evaluation of Indeterminate
forms.
Partial derivatives – Definition and simple problems, Euler‟s
theorem(without proof) – problems, total derivatives, partial

DEPT OF MATHS, SJBIT Page 1


ENGINEERING MATHEMATICS-I 15MAT11

differentiation of composite functions-problems, Jacobians-


definition and problems . 10hrs
Module –3
Vector Calculus:
Derivative of vector valued functions, Velocity, Acceleration and
related problems, Scalar and Vector point functions.Definition
Gradient, Divergence, Curl- problems . Solenoidal and Irrotational
vector fields. Vector identities - div ( F A), curl ( F A),curl (grad F
), div (curl A). 10hrs

Module- 4
Integral Calculus:
Reduction formulae ∫ sinnx dx ∫cosnx dx ∫sinnxcosmxdx,, (m and n
are positive integers), evaluation of these integrals with standard
limits (0 to л/2) and problems.
Differential Equations:
Solution of first order and first degree differential equations –
Exact, reducible to exact and Bernoulli‟s differential equations.
Applications- orthogonal trajectories in Cartesian and polar forms.
Simple problems on Newton‟s law of cooling.. 10hrs

Module –5
Linear Algebra Rank of a matrix by elementary transformations,
solution of system of linear equations - Gauss- elimination method,
Gauss- Jordan method and Gauss-Seidel method. Rayleigh‟s
power method to find the largest Eigen value and the
corresponding Eigen vector. Linear transformation, diagonalisation
of a square matrix, Quadratic forms, reduction to Canonical form
10hrs

DEPT OF MATHS, SJBIT Page 2


ENGINEERING MATHEMATICS-I 15MAT11

CONTENTS

 Module I : Differential Calculus- I…………….4 - 48

 Module II : Differential Calculus- II……………49 -81

 Module III: Vector Calculus………………........82-104

 Module IV : Integral Calculus…………………105-125

 Module V : Linear Algebra ……………………126-165

DEPT OF MATHS, SJBIT Page 3


ENGINEERING MATHEMATICS-I 15MAT11

MODULE I
DIFFERENTIAL CALCULUS-I
CONTENTS:
 Successive differentiation …………………………………………..3

 nth derivatives of some standard functions…………………...7

 Leibnitz‟s theorem (without proof)………………………..…16


 Polar curves

Angle between Polar curves…………………………………….20

Pedal equation for Polar curves………………………………...24

Derivative of arc length………………………………………….28

 Radius of Curvature……………………………………………….34

 Expression for radius of curvature in case of Cartesian Curve …35

 Expression for radius of curvature in case of Parametric

Curve………………………………………………………………..36

 Expression for radius of curvature in case of Polar Curve……...41

 Expression for radius of curvature in case of Pedal Curve……...43

DEPT OF MATHS, SJBIT Page 4


ENGINEERING MATHEMATICS-I 15MAT11

SUCCESSIVE DIFFERENTIATION
In this lesson, the idea of differential coefficient of a function and its successive
derivatives will be discussed. Also, the computation of nth derivatives of some
standard functions is presented through typical worked examples.

1. Introduction:- Differential calculus (DC) deals with problem of calculating rates of


change. When we have a formula for the distance that a moving body covers as a
function of time, DC gives us the formulas for calculating the body‟s velocity and
acceleration at any instant.
 Definition of derivative of a function y = f(x):-

f ( x  x)  f ( x)
Fig.1. Slope of the line PQ is
x
The derivative of a function y = f(x) is the function f (x) whose value at each x is
defined as
dy
= f (x) = Slope of the line PQ (See Fig.1)
dx
f ( x  x)  f ( x)
= lim -------- (1)
x  0 x
= lim (Average rate change)
x  0

= Instantaneous rate of change of f at x provided the limit exists.

The instantaneous velocity and acceleration of a body (moving along a line) at any instant
x is the derivative of its position co-ordinate y = f(x) w.r.t x, i.e.,
dy
Velocity = = f (x) --------- (2)
dx
And the corresponding acceleration is given by

d2y
Acceleration   f ( x) ---------- (3)
dx 2

DEPT OF MATHS, SJBIT Page 5


ENGINEERING MATHEMATICS-I 15MAT11

Successive Differentiation:-
The process of differentiating a given function again and again is called as
Successive differentiation and the results of such differentiation are called
successive derivatives.
 The higher order differential coefficients will occur more frequently in spreading
a function all fields of scientific and engineering applications.
 Notations:
dy d 2 y d 3 y dny
i. , 2 , 3 ,…….., nth order derivative:
dx dx dx dx n
ii f (x) , f (x) , f (x) ,…..., nth order derivative: f n (x)
iii Dy, D 2 y , D 3 y ,………..., nth order derivative: D n y
iv y  , y  , y  ,……, nth order derivative: y (n )
v. y1 , y 2 , y 3 …, nth order derivative: y n
 Successive differentiation – A flow diagram
df
Input function: y  f (x) Operation Output function y   f (x) (first order
d dx
dx
derivative)

d2 f
Input function y   f (x) Operation Output function y    f ( x) (second order
d
dx
dx 2
derivative)

d3 f
Input function y   f (x) Operation Output function y    f ( x) (third order
d
dx
dx 3
derivative)

------------------------------------------------------------------------------------------------------------
dn f
Input function y n1  f n1 ( x) Operation Output function y n  n
 f n (x) (nth order
d
dx
dx
derivative)

DEPT OF MATHS, SJBIT Page 6


ENGINEERING MATHEMATICS-I 15MAT11

Calculation of nth derivatives of some standard functions


 Below, we present a table of nth order derivatives of some standard functions for
ready reference.
Sl. y = f(x) dny
No yn  n  D n y
dx
1 e mx n mx
m e
2 a mx m n log a  a mx
n

3 ax  bm i. mm  1m  2....m  n  1a n ax  b


mn
for all m .
ii. 0 if m  n
iii. n! a n if m  n
m!
iv. x mn if m  n
m  n !
4 1 (1) n n ! n
ax  b  (ax  b) n 1
a

5. 1 (1) n (m  n  1) ! n
a
ax  b m (m  1) !(ax  b) m n
6. log( ax  b) (1) n 1 (n  1) ! n
a
(ax  b) n
7. sin(ax  b) a n sin(ax  b  n  )
2
8. cos(ax  b) a n cos(ax  b  n  )
2
9. e ax sin(bx  c) r n e ax sin(bx  c  n ) , r  a 2  b 2   tan 1 ( b a)
10. e ax cos(bx  c) r n e ax cos(bx  c  n ) , r  a 2  b 2   tan 1 ( b a)
 We proceed to illustrate the proof of some of the above results, as only the
above functions are able to produce a sequential change from one derivative to
the other. Hence, in general we cannot obtain readymade formula for nth
derivative of functions other than the above.

1. Consider e mx . Let y  e mx . Differentiating w.r.t x, we get


y1  memx . Again differentiating w.r.t x, we get
 
y 2  m me mx = m 2e mx
Similarly, we get
y3 = m 3 e mx
y4 = m 4e mx
…………….
And hence we get

DEPT OF MATHS, SJBIT Page 7


ENGINEERING MATHEMATICS-I 15MAT11

y n = m n e mx 
dx n
 
d n mx
e  m n e mx .

2. ax  bm
let y  ax  b  Differentiating w.r.t x,
m

y1 = m ax  b a . Again differentiating w.r.t x, we get


m 1

y2 = m m  1 ax  b a 2
m 2

Similarly, we get
y3 = m m  1 m  2 ax  b a 3
m3

………………………………….
And hence we get
y n = m m  1 m  2 ………. m  n  1 ax  b a n for all m.
mn

Case (i) If m  n (m-positive integer),then the above expression becomes


y n = n n  1 n  2 ……….3.2.1 ax  b a n
nn

i.e. y n  n! a n
Case (ii) If m<n,(i.e. if n>m) which means if we further differentiate the above
expression, the

right hand site yields zero. Thus D n ax  b  0 if m  n
m

If m>n, then y n  mm  1m  2......m  n  1ax  b a n becomes
mn
Case (iii)
mm  1m  2......m  n  1m  n !
 ax  bmn a n
m  n!
. i.e yn 
m!
ax  bmn a n
m  n!
1
3.
ax  bm
 ax  b 
1 m
Let y
ax  b m

Differentiating w.r.t x
y1  max  b a   1max  b
 m1  m1
a

y 2   1m  m  1ax  b
 m11

a   1 mm  1ax  b
2  m 2 
a2
y3   1 mm  1m  2ax  b
3  m3  3
Similarly, we get a
y 4   1 mm  1m  2m  3ax  b
4  m 4 
a4
……………………………
y n   1 mm  1m  2.....m  n  1ax  b
n  m n  n
a
This may be rewritten as

yn 
 1 m  n  1m  n  2.....m  1mm  1!
n
ax  bmn  a n
m  1!

DEPT OF MATHS, SJBIT Page 8


ENGINEERING MATHEMATICS-I 15MAT11

or yn 
 1 m  n  1! n
n
a
m  1!ax  bmn
1
4.
ax  b 
Putting m  1, in the result
 1  (1) n (m  n  1) ! n
D  n
m 
 m n
a
 (ax  b)  (m  1) !(ax  b)
 1  (1) n (1  n  1) ! n
we get D n   1 n
a
 (ax  b)  (1  1) !(ax  b)
 1  (1) n n ! n
or D n    1 n
a
 (ax  b)  (ax  b)

Find the nth derivative of the following examples

1. (a) log( 9 x 2  1) 
(b) log (4 x  3)e 5 x7  (c) log 10
(3x  5) 2 (2  3x)
( x  1) 6
Sol: (a) Let y  log( 9 x 2  1)  log(3x  1)(3x  1)
y  log( 3x  1)  log( 3x  1) ( log( AB)  log A  log B )

 y n  n log( 3x  1)  n log( 3x  1)


dn dn
dx dx
(1) (n  1) ! n (1) n 1 (n  1) ! n
n 1
i.e y n  (3)  (3)
(3x  1) n (3x  1) n

 
(b) Let y  log (4 x  3)e 5 x7  log( 4 x  3)  log e 5 x7
 log( 4 x  3)  (5x  7) log e e ( log A B  B log A )
 y  log( 4 x  3)  (5x  7) ( log e e  1 )
(1) n 1 (n  1) ! n
 yn  (4)  0  D(5x  6)  5
(4 x  3) n
D 2 (5x  6)  0
D n (5x  1)  0 (n  1)

(3x  5) 2 (2  3x)
(c) Let y  log 10
( x  1) 6
1   (3x  5) (2  3x) 
2
 log e X
    log 10 X 
log e 10 
 ( x  1) 6

 log e 10

DEPT OF MATHS, SJBIT Page 9


ENGINEERING MATHEMATICS-I 15MAT11

1 1  (3x  5) 2 (2  3x) 
  log    log A B  B log A
log e 10  2  ( x  1) 6 
 A
 log   log A  log B
B

1
2 log e 10

log( 3x  5) 2  log( 2  3x)  log( x  1) 6 
 y
1
2 log( 3x  5)  log( 2  3x)  6 log( x  1)
2 log e 10
Hence,

1  (1) n1 (n  1) ! n (1) n1 (n  1) ! (1) n1 (n  1) ! n 


yn  2. (3)  ( 3) n
 6. (1) 
2 log e 10  (3x  5) n (2  3 x) n ( x  1) n 

2. (a) e 2 x 4  6 2 x4 (b) cosh 4 x  cosh 2 4 x


1 1
(c) e  x sinh 3x cosh 2 x (d)   (6 x  8) 5
(4 x  5) (5 x  4) 4

Sol: (a) Let y  e 2 x 4  6 2 x 4


 e2x e4  62x 64
 y  e 4 (e 2 x )  1296(6 2 x )
dn dn
hence y n  e 4 n (e 2 x )  1296 n (6 2 x )
dx dx
  
 e 2 e  1296 2 (log 6) n 6 2 x
4 n 2x n

(b) Let y  cosh 4 x  cosh 2 4 x
2
 e 4 x  e 4 x   e 4 x  e 4 x 
     
 2   2 

2

1 4x
  1
 e  e 4 x  (e 4 x ) 2  (e 4 x ) 2  2(e 4 x )(e 4 x )
4

2

1 4x
  1
y  e  e 4 x  e 8 x  e 8 x  2
4

hence,
1
2
  
1
y n  4 n e 4 x  (4) n e 4 x  8 n e 8n  (8) n e 8n  0
4

x
(c) Let y  e sinh 3x cosh 2 x
 e 3 x  e 3 x  e 2 x  e 2 x 
 e x   
 2  2 
x

e
4
(e 3x  e 3x )(e 2 x  e 2 x )
DEPT OF MATHS, SJBIT Page 10
ENGINEERING MATHEMATICS-I 15MAT11

ex 5x

4

e  e  x  e x  e 5 x 
 e  e 2 x  1  e 6 x 
1 4x
4
y  1  e 4 x  e 2 x  e 6 x 
1
4
y n  0  (4) n e 4 x  (2) n e 2 x  (6) n e 6 x 
1
Hence,
4
1 1
(d) Let y    (6 x  8) 5
(4 x  5) (5 x  4) 4

Hence, yn 
dn 1  dn 
  n 
1  dn
4

 n 6 x  8
5

 (4 x  5)  dx  (5 x  4)  dx
dx n
(1) n n! (1) n (4  n  1)!
 n 1
( 4) n
 4 n
(5) n  0
(4 x  5) (4  1)!(5 x  4)
(1) n n ! (1) n (3  n) ! n
i.e y n  ( 4) n
 (5)
(4 x  5) n 1 3!(5 x  4) n 4
Evaluate
1 1 x2
1. (i) (ii) (iii)
x 2  6x  8 1  x  x2  x3 2x 2  7x  6

 x  2
(iv)   2
1
 x  1  4 x  12 x  9
 a  (vi) tan
(v) tan 1 x 1 1 x 
x (vii) tan 1  
1 x 

1 1
Sol: (i) Let y  . The function can be rewritten as y 
x  6x  8
2
( x  4)( x  2)
This is proper fraction containing two distinct linear factors in the denominator.
So, it can be split into partial fractions as

1 A B
y   Where the constant A and B are found
( x  4)( x  2) ( x  4) ( x  2)

as given below.

1 A( x  2)  B( x  4)

( x  4)( x  2) ( x  4)( x  2)

 1  A( x  2)  B( x  4) -------------(*)

Putting x = 2 in (*), we get the value of B as B   1


2

DEPT OF MATHS, SJBIT Page 11


ENGINEERING MATHEMATICS-I 15MAT11

Similarly putting x = 4 in (*), we get the value of A as A  1


2

1 (1 / 2) (1 / 2)
y   Hence
( x  4)( x  2) x  4 x2
1 dn  1  1 dn  1 
yn     
2 dx n  x  4  2 dx n  x  2 

1  (1) n n ! n 1  (1) n n ! 
  n 1
(1)    n 1
(1) n 
2  ( x  4)  2  ( x  2) 

1  1 1 
 (1) n n !  n 1
 n 1 
2  ( x  4) ( x  2) 

1 1 1
(ii) Let y   
1 x  x  x
2 3
(1  x)  x (1  x) (1  x)(1  x 2 )
2

1 1
ie y  
(1  x)(1  x)(1  x) (1  x) 2 (1  x)
Though y is a proper fraction, it contains a repeated linear factor (1  x) 2
in its
denominator. Hence, we write the function as
A B C
y   in terms of partial fractions. The constants
(1  x) (1  x) 2
1 x
A, B, C
are found as follows:

1 A B C
y   
(1  x) (1  x) (1  x) (1  x)
2 2
1 x

ie 1  A(1  x)(1  x)  B(1  x)  C (1  x) 2 -------------(**)


Putting x = 1 in (**), we get B as B  1
2
Putting x = -1 in (**), we get C as C  1
4
Putting x = 0 in (**), we get 1  A  B  C
 A  1 B  C  1 1  1  1
2 4 4
A 1
4
(1 / 4) (1 / 2) (1 / 4)
Hence, y   
(1  x) (1  x) 2
(1  x)

DEPT OF MATHS, SJBIT Page 12


ENGINEERING MATHEMATICS-I 15MAT11

1  (1) n n ! n  1  (1) n (2  n  1) ! n  1  (1) n n ! n 


 yn   (1)    (1)    (1) 
4  (1  x) n1  2  (2  1) !(1  x)
2 n
 4  (1  x)
n 1

1  1 1  1  (1) n (n  1) !
 (1) n n !  n 1
 n 1 
  
4  (1  x ) (1  x )  2  (1  x)n  2 
x2
(iii) Let y  2 (VTU July-05)
2x  7 x  6
This is an improper function. We make it proper fraction by actual division
and later
spilt that into partial fractions.
1 ( 7 x  3)
i.e x 2  (2 x 2  7 x  6)   2 2
2 2x  7 x  6
2 x3
1 7
y  Resolving this proper fraction into partial fractions,
2 (2 x  3)( x  2)

we get
1  A B 
y  
2  (2 x  3) ( x  2) 
. Following the above examples for finding A &

B, we get
1  92 (4) 
y  
2  2 x  3 x  2 
9  (1) n n ! n  (1) n n ! 
Hence, y n  0   n 1
( 2) 
  4 n 1
(1) n 
2  (2 x  3)   ( x  2) 
 9 (2) n 4 
i.e y n  (1) n n !  2 n 1
 n 1 
 (2 x  3) ( x  2) 
( x  2) x
(iv) Let y   2
( x  1) 4 x  12 x  9

(i) (ii)
Here (i) is improper & (ii) is proper function. So, by actual division (i)
becomes
 x  2  1 
   1   . Hence, y is given by
 x 1   x  1

 1  1
y  1   [ (2 x  3) 2  4 x 2  12 x  9 ]
 x  1  ( 2 x  3) 2

Resolving the last proper fraction into partial fractions, we get


x A B
  . Solving we get
(2 x  3) 2
(2 x  3) (2 x  3) 2

DEPT OF MATHS, SJBIT Page 13


ENGINEERING MATHEMATICS-I 15MAT11

A 1 and B   3
2 2
 1   12  32 
 y  1    2 
 1  x   (2 x  3) (2 x  3) 
 (1) n n ! n  1  (1) n n ! n 3  (1) n (n  1) ! n 
 yn  0   (1)    n 1
( 2)    (2) 
 (1  x)  2  (2 x  3)  2  (2 x  3)n  2
n

(v) tan 1 x  a
Let y  tan 1 x  a
1 1 a
 y1    2
1 x  a 2
a x a
2

 a 
y n  D n y  D n1 ( y1 )  D n1  2 2 
x a 
a a
Consider 2 
x a 2
( x  ai)( x  ai)
A B
  , on resolving into partial fractions.
( x  ai) ( x  ai)


1   
2 i 
1
2i , on solving for A & B.
( x  ai ) ( x  ai )
 a    1 2i  n 1 
1
2i 
 D n1  2 2 
 D n1  D  
x a   x  ai   x  ai 
 1   (1) (n  1) !  1  (1) (n  1) !
n 1 n 1
          -----------(*)
 2i   ( x  ai)   2i  ( x  ai)
n n

a
We take transformation x  r cos  a  r sin  where r  x 2  a 2 ,   tan 1  
 x
x  ai  r cos   i sin    rei
x  ai  r cos   i sin    re i
1 1 e in 1 e in
  , 
x  ai n r n e in r n x  ai n rn

now(*) is y n 
 1 n  1! in
n 1
e  e in 
2 i rn

yn 
 1n1 2 i sin n    1n1 n  1! sin n
2 i rn rn

DEPT OF MATHS, SJBIT Page 14


ENGINEERING MATHEMATICS-I 15MAT11

(vi) Let y  tan 1 x .Putting a = 1 in Ex.(v) we get


y n which is same as above with r  x 2  1   tan 1 1  x
  cot 1 x  or x  cot 
1 1
 r  cot 2   1  cos ec    sin n 
r n
cos ec n
 
D n tan 1 x   1 n  1 ! sin n  sin n where   cot 1 x
n 1

1 x 
(vii) Let y  tan 1  
1 x 
put x  tan   tan 1 x
1  tan  
 y  tan 1 
1  tan  
 tan 1 tan( 4   ) 
 tan 
4
  1  tan  
   
 1  tan  

    tan 1 ( x)
4 4

y  tan 1 ( x)
4
y n  0  D n (tan 1 x)
 1   (1) (n  1) !  1  (1) (n  1) !
n 1 n 1
    
   
 2i   ( x  ai)   2i  ( x  ai)
n n

nth derivative of trigonometric functions:

1. sin(ax  b) .
Let y  sin(ax  b) . Differentiating w.r.t x,

y1  cos(ax  b).a As sin( x  )  cos x
2
We can write y1  a sin(ax  b   / 2).
again differentiating w.r.t x, y 2  a cos(ax  b   / 2).a

Again using sin( x  )  cos x ,we get y 2 as
2
y 2  a sin(ax  b   / 2   / 2).a
i.e. y 2  a 2 sin(ax  b  2 / 2).
Similarly, we get
y3  a 3 sin(ax  b  3 / 2).
y 4  a 4 sin(ax  b  4 / 2).
y n  a n sin(ax  b  n / 2).

DEPT OF MATHS, SJBIT Page 15


ENGINEERING MATHEMATICS-I 15MAT11

2. e ax sinbx  c .
Let y  eax sinbx  c .....(1)
Differentiating using product rule ,we get
y1  e ax cosbx  c b  sinbx  c  ae ax
y1  e a sin bx  c   b cosbx  c   . For computation of higher order
ax
i.e.
derivatives
it is convenient to express the constants „a‟ and „b‟ in terms of the
constants r and
 defined by a  r cos  & b  r sin  ,so that
 
r  a 2  b 2 and   tan 1 b .thus,
a
y1 can be rewritten as
y1  e ax r cos   sinbx  c   r sin   cosbx  c  
or y1  e ax r{sinbx  c cos   cosbx  c  cos  }
i.e. y1  re ax sinbx  c   ...........(2)
Comparing expressions (1) and (2), we write y 2 as
y 2  r 2 e ax sinbx  c  2 
y3  r 3 e ax sinbx  c  3 
Continuing in this way, we get
y 4  r 4 e ax sinbx  c  4 
y5  r 5 e ax sinbx  c  5 
…………………………….
y n  r n e ax sinbx  c  n 
 
 D n e ax sinbx  c   r n e ax sinbx  c  n , where
r  a 2  b 2 &   tan 1 b
a
 
Solve the following:
1. (i) sin 2 x  cos 3 x (ii) sin 3 cos 3 x (iii) cos x cos 2 x cos 3x
(iv) sin x sin 2 x sin 3x (v) e 3 x cos 2 x 
(vi) e 2 x sin 2 x  cos 3 x 
The following formulae are useful in solving some of the above problems.
1  cos 2 x 1  cos 2 x
(i) sin 2 x  (ii ) cos 2 x 
2 2
(iii) sin 3x  3 sin x  4 sin 3 x (iv ) cos 3x  4 cos 3 x  3 cos x
(v) 2 sin A cos B  sin A  B  sin A  B
(vi) 2 cos A sin B  sin A  B  sin A  B
(vii) 2 cos A cos B  cos A  B  cos A  B

DEPT OF MATHS, SJBIT Page 16


ENGINEERING MATHEMATICS-I 15MAT11

(viii) 2 sin A sin B  cos A  B  cos A  B

 1  cos 2 x  1
Sol: (i) Let y  sin 2 x  cos 3 x     cos 3x  3 cos x 
 2  4
    
 y n  0  2 cos 2 x  n  3 cos 3x  n  3 cos x  n
1
2
n
2 4
1 n
2 2
  
sin 3 2 x 1   sin 6 x  3 sin 2 x 
3
 sin 2 x 
(ii)Let y = sin 3 x cos 3 x       
 2  8 8 4
 3 sin 2 x  sin 6 x
1
32
1  n  n  n  n 
yn  3.2 sin 2 x    6 sin 6 x  
32   2   2 
(iii) )Let y = cos 3x cos x cos 2 x
1
2
1
2

= cos 4 x  cos 2 x  cos 2 x  cos 4 x cos 2 x  cos 2 2 x 
1 1 1  cos 4 x 
=  cos 6 x  cos 2 x   
2 2 2
 1  cos 4 x 
1 cos 2 x 1
 cos 6 x 
4 4 4
 n   n 
2 n cos 2 x   4 cos 4 x 
n

1 n  n   2   2 
 y n  6 cos 6 x   
4  2  4 4
(iv) )Let y = sin 3x sin x sn2 x

 sin 2 x   sin 4 xsin 2 x


1
2
1

 sin 2 2 x  sin 4 x sin 2 x
2

1 1  cos 4 x 1 
=   sin 2 x  sin 6 x 
2 2 2 
 1  cos 4 x  1 
    sin 2 x  sin 6 x 
 4  4 
1 n  n   n   n 
yn   4 cos 4 x    2 sin 2 x 
n
  6 sin 6 x 
n

4  2   2   2 

(v) Let y  e 3 x cos 2 x


 y n  re3 x cos2 x  n 
2
where r  32  2 2  13 &   tan 1  
3

DEPT OF MATHS, SJBIT Page 17


ENGINEERING MATHEMATICS-I 15MAT11


(vi) Let y = e 2 x sin 2 x  cos 3 x 

We know that sin 2 x  cos 3 x 
1  cos 2 x 1
2

 cos 3x  3 cos x
4
 
1  cos 2 x  e
2x
 y = e 2 x sin 2 x  cos 3 x  e 2 x    4 cos 3x  3 cos x
 2 
1
2
 1
 
 y  e 2 x  e 2 x cos 2 x  e 2 x cos 3x  3e 2 x cos x
4

Hence,
1
2
 1
4
  
y n  2 n e 2 x  r1n e 2 x cos2 x  n1   r2n e 2 x cos3x  n 2   3r3n e 2 x cosx  n 3 

where r1  2 2  2 2  8 ; r2  2 2  32  13 ; r3  2 2  12  5
2 3 1
1  tan 1   ; 2  tan 1   ; 3  tan 1   ;
2 2 2

Leibnitz’s Theorem

Leibnitz‟s theorem is useful in the calculation of nth derivatives of product of two


functions.

Statement of the theorem:


If u and v are functions of x, then
D (uv)  D n uv nC1 D n1uDv nC2 D n2 uD 2 v  .... nCr D nr uD r v  ...uD n v ,
n

d n nn  1 n!
where D  , C1  n , n C 2  ,........, n C r 
dx 2 r!n  r !

Examples
1. If x  sin t , y  sin pt prove that
 
1  x 2 y n2  2n  1xy n1  p 2  n 2 yn  0  
Solution: Note that the function y  f (x) is given in the parametric form with a
parameter t.
So, we consider
dy dy dt p cos pt
  (p – constant)
dx dx dt cos t
p 2 cos 2 pt p 2 (1  sin 2 pt ) p 2 (1  y 2 )
2
 dy 
or     
 dx  cos 2 t 1  sin 2 t 1 x2
 
or 1  x 2 y12  p 2 1  y 2  
 
So that 1  x 2 y12  p 2 1  y 2  
Differentiating w.r.t. x,
 
1  x 2 y1 y 2   y1  2 x   p  2 yy1   0
2 2 2

DEPT OF MATHS, SJBIT Page 18
ENGINEERING MATHEMATICS-I 15MAT11

1  x y
 xy1  p 2 y  0
2
2 --------------- (1) [  2y1 , throughout]
Equation (1) has second order derivative y 2 in it. We differentiate (1), n times,
term wise,
using Leibnitz‟s theorem as follows.
 
D n 1  x 2 y 2  xy1  p 2 y  0 
 
i.e D n (1  x 2 ) y 2  D n xy1   D n p 2 y  0  
---------- (2)

(a) (b) (c)


Consider the term (a):
  
D n 1  x 2 y 2 . Taking u  y 2 and v  (1  x 2 ) and applying Leibnitz‟s theorem
we get
D n uv  D n uv nC1 D n1uDv nC2 D n2 D 2 v nC3 D n3uD 3 v  ...
i.e
 
D n y 2 (1  x 2 )  D n ( y 2 ).(1  x 2 ) nC1 D n1 ( y 2 ).D(1  x 2 ) nC2 D n2 ( y 2 ) D 2 (1  x 2 ) nC3 D n3 ( y2 ) D 3 (1  x 2 ) 

n(n  1) n(n  1)(n  2)


 y ( n ) 2  x 2 )  ny ( n1) 2 .(2 x)  y( n2) 2 .(2)  . y ( n3) 2 .(0)  ...
2! 3!
    
D n 1  x 2 y 2  1  x 2 y n2  2nxy n1  n(n  1) y n ----------- (3)
Consider the term (b):
D n xy1  . Taking u  y1 and v  x and applying Leibnitz‟s theorem,
we get
D n y1 ( x)  D n ( y1 ).( x) nC1 D n1 y1 .D( x) nC2 D n2 ( y1 ).D 2 ( x)  ....
n(n  1)
 y ( n )1 .x  ny ( n1)1  y ( n2) 2 (0)  ....
2!
D n xy1   xy n1  ny n ---------- (4)
Consider the term (c):
D n ( p 2 y)  p 2 D n ( y)  p 2 y n --------- (5)
Substituting these values (3), (4) and (5) in Eq (2) we get
1  x 2 yn2  2nxy n1  n(n  1) yn  xy n1  nyn  p 2 yn   0
ie 1  x 2 y n2  (2n  1) xy n1  n 2 y n  ny n  ny n  p 2 y n  0
 1  x 2 y n2  (2n  1) xy n1   p 2  n 2 y n  0 as desired.

2. If sin 1 y  2 log( x  1) or y  sin2 log( x  1) or y  sin log( x  1) 2 or  


y  sin log( x  2 x  1) , show that x  1 y n 2  2n  1x  1y n1  n 2  4 y n  0
2 2
 
(VTU Jan-03)
Sol: Out of the above four versions, we consider the function as
sin 1 ( y)  2 log( x  1)
Differentiating w.r.t x, we get

DEPT OF MATHS, SJBIT Page 19


ENGINEERING MATHEMATICS-I 15MAT11

1  2 
( y1 )    ie ( x  1) y1  2 1  y
2

1 y 2
 x  1 
Squaring on both sides
x  12 y12  4(1  y 2 )
Again differentiating w.r.t x,
x  12 2 y1 y2   y12 2( x  1)  4(2 yy1 )
or x  12 y2  ( x  1) y1  4 y (2 y1 )
or x  12 y2  ( x  1) y1  4 y  0 -----------*
Differentiating * w.r.t x, n-times, using Leibnitz‟s theorem,
 n n(n  1) n2 
n 1
D y 2 ( x  1)  nD ( y 2 )2( x  1) 
2

D ( y 2 )(2)  D n ( g1 )( x  1)  nD n1 y1 (1)  4 D n y  0 
 2! 
On simplification, we get
x  12 yn2  2n  1x  1yn1  n 2  4 yn  0  
3. If x  tan(log y) , then find the value of
1  x 2 yn1  2nx  1yn  n(n  1) yn1 (VTU July-04)
Sol: Consider x  tan(log y)
1
i.e. tan 1 x  log y or y  e tan x
Differentiating w.r.t x,
1 1 y
y1  e tan x . 
1 x 2
1 x2

 1  x 2 y1  y  
ie 1  x 2 y1  y  0 -----------* 
We differentiate * n-times using Leibnitz‟s theorem,
We get
  
D n 1  x 2 y1  D n ( y)  0
ie.
D ( y1 )(1  x 2 ) nC1 D n1 ( y1 ) D(1  x 2 ) nC2 D n2 ( y1 ) D 2 (1  x 2 )  ....  D n y  0
n
  
 n(n  1) 
ie  y n1 (1  x 2 )  ny n (2 x)  y n1 (2)  0  ....  y n  0
 2! 
1  x yn1  2nx  1yn  n(n  1) yn1  0
2

4. If y m  y  m  2 x , or y  x  x 2  1 or y  x  x 2  1
1 1
 
m
  m

Show that x 2  1y n2  (2n  1) xy n1  n 2  m 2 y n  0 (VTU Feb-02)

1
 y  2x    2x
1 1 1
Sol: Consider y m m
y m
1
ym
 y 1
m
2
 2x y    1  0 Which is quadratic equation in y
1
m
1
m

DEPT OF MATHS, SJBIT Page 20


ENGINEERING MATHEMATICS-I 15MAT11

 (2 x)  (2 x) 2  4(1)(1) 2 x  4 x 2  4


y  
1
m

2(1) 2


2x  2 x 2  1
2
 
 x  x2 1  y m  x  x2 1
1
 

 y  x  x2 1 
m

so, we can consider y  x  x 2  1 


m
or 
y  x  x2 1 
m


Let us take y  x  x 2  1  m


 y1  m x  x 2  1

1 

1

m 1
( 2 x )



 2 x 1 
2

 m 1 
y1  m x  x 2  1 


x 2  1  x 
x 2  1 

or
 x  1y  my . On squaring
2
1

x 2

 1 y12  m 2 y 2 .

Again differentiating w.r.t x,

x 2

 1 2 y1 y 2  y12 (2 x)  m 2 (2 yy1 )
or
x 2

 1 y 2  xy1  m 2 y (2 y1 )
or
x 2

 1 y 2  xy1  m 2 y  0 ------------(*)

Differentiating (*) n- times using Leibnitz‟s theorem and simplifying, we get


x 2  1yn2  (2n  1) xy n1  n 2  m2 yn  0
POLAR CURVES
Angle between Polar Curves:
Introduction:- We are familiar with Cartesian coordinate system for specifying a point
in the xy – plane. Another useful system for similar purpose is Polar coordinate system,
and the curves specified by these coordinates are referred to as polar curves.
 A polar curve by name “three-leaved rose” is displayed below:

DEPT OF MATHS, SJBIT Page 21


ENGINEERING MATHEMATICS-I 15MAT11


ө=
2

3 
ө= ө=
4 4

ө=π ө=0

3
ө=
2

 Any point P can be located on a plane with co-ordinates r , 


called polar co-ordinates of P where r = radius vector OP,(with pole „O‟)
 = projection of OP on the
initial axis OA.(See Fig.)
 The equation r  f   is known as a polar curve.
 Polar coordinates r ,  can be related with Cartesian coordinates x, y  through
the relations
 Fig.1. Polar coordinate system
x  r cos  & y  r sin  .

Theorem 1: Angle between the radius vector and the tangent:


Y T

d
i.e., With usual notation prove that tan   r L
dr φ
P(r, ө)
 Proof:- Let “  ” be the angle between the radius vector OPL φ
and the tangent TPT 1 at the point `P` on the polar r
ψ
Ө
curve r  f   . (See fig.2) O T
A
r = f(ө)
From Fig.2, Fig.2. Angle between radius
   vector and the tangent
tan   tan  1
tan  tan     1
1  tan  tan  1

dy tan   tan 
i.e.  ................. (1)
dx 1  tan  tan 

DEPT OF MATHS, SJBIT Page 22


ENGINEERING MATHEMATICS-I 15MAT11

On the other hand, we have x  r cos  ; y  r sin  differentiating these,


w.r.t  ,
 dr   dr 
 r  sin    cos    r cos    sin  
dx dy
& 
d  d  d  d 
 dr 
dy r cos    sin   
dy d  d  dr
  dividing the Nr & Dr by cos 
dx dx  dr  d
d r  sin    cos   
 d 
dy

 r d
dr
  tan 
dx  rd dr  tan   1

dy tan   r dr
d
i.e. 

dx 1  tan  rd  dr
………………….(2)

Comparing equations (1) and (2)


we get tan   r d
dr
 1 dr 
 Note that cot    
 r d 
 A Note on Angle of intersection of two polar curves:-
If 1 and  2 are the angles between the common radius vector and the tangents
at the point of intersection of two curves r  f1   and r  f 2   then the
angle intersection of the curves is given by 1  2
Theorem 2: The length “p” of perpendicular from pole to the tangent in a polar curve
1 1 1  dr  2
i.e.(i) p  r sin  or (ii)  2  4 
p 2
r r  d 
Proof:- In the Fig.3, note that ON = p, the length of the perpendicular from the pole to
the tangent at p on r  f   .from the right angled triangle OPN,

 ON  OPsin 
ON
sin  
OP
φ
i.e. p  r sin ..............(i) P(r, ө)

1 1 1
r φ Ψ
Consider   cos ec O Ө
p r sin  r
r = f (ө)
 2  2 cos ec 2  2 1  cot 2 
1 1 1
p r r P

1   1 dr   N
2
1
 1     Fig.3 Length of the perpendicular
p 2 r 2   r d  
from the pole to the tangent

DEPT OF MATHS, SJBIT Page 23


ENGINEERING MATHEMATICS-I 15MAT11

2
1 1 1  dr 
  2  4   ............(ii )
 d 
2
p r r
2
1 1  du 
Note:-If u  , we get 2  u 2   
r p  d 

In this session, we solve few problems on angle of intersection of polar curves and pedal
equations.
Examples:-
Find the acute angle between the following polar curves
1. r  a1  cos   and r  b1  cos   (VTU-July-2003)

2 r  sin   cos   and r  2 sin  (VTU-July-2004)


3. r  16 sec 2  2  and r  25 cos ec  2  2

4. r  a log  and r  a (VTU-July-2005)


log 
a a
5. r  and r 
1 1 2
Sol:

1. Consider Consider
r  a1  cos   r  b1  cos  
Diff w.r.t  Diff w.r.t 

dr dr
 a sin   b sin 
d d
d a 1  cos   d b1  cos  
r  r 
dr  a sin  dr b sin 

tan 1  
 
2 cos 2 
2 tan 1  
 
2 sin 2 
2
2 sin    
2
cos 
2
2 sin    
2
cos 
2
  cot   tan 
2 2
i.e tan 1  tan 
2

   1    
2 2

2
  tan 1  tan 
2
 1  2
Angle between the curves
 
1  2   2   2   2   2
Hence, the given curves intersect orthogonally.

2. Consider Consider
r  sin   cos   r  2 sin 
Diff w.r.t  Diff w.r.t 

DEPT OF MATHS, SJBIT Page 24


ENGINEERING MATHEMATICS-I 15MAT11

dr dr
 cos   sin   2 cos 
d d
d sin   cos  d 2 sin 
r  r 
dr cos   sin  dr 2 cos 
tan   1
tan 1  (÷ Nr & Dr cos  ) tan 2  tan 
1  tan 
i.e tan 1 
tan   1
1  tan 
 tan   
4
   2  

 1   
4
 Angle between the curves = 1  2    4

   
4

3. Consider Consider
r  16 sec  2
2
  r  25 cos ec 2   2
Diff w.r.t  Diff w.r.t 
dr
d
   
 32 sec 2  tan  . 1
2 2 2
dr
d
   
 50 cos ec 2  cot  . 1
2 2 2
 2  2
 16 sec  tan     
 25 cos ec 2  cot 
2 2
d 16 sec   2
d 25 cos ec   2

 2  2
dr 16 sec   tan  dr  25 cos ec  cot  
r r
 2  2
2 2 2 2
tan   cot   tan    tan    tan   tan  
1 2 2 2 2 2 2

 1    2

2
  2  
2
Angle of intersection of the curves = 1  2    2

2
  2 

2

4. Consider Consider
r  a log  r a
log 
Diff w.r.t  Diff w.r.t 
  a log   . 1
dr a dr

2

d  d 
 log  2  
r
d
dr
 a log  
a
  r
d
dr
  a 

 log   a



tan 1   log  ..........(i) tan 2   log  ..........(ii )
We know that

DEPT OF MATHS, SJBIT Page 25


ENGINEERING MATHEMATICS-I 15MAT11

tan 1  tan  2
tan1   2  
1  tan 1 tan  2

 log     log  

1   log    log  
2 log 
i.e tan1   2   ............(iii )
1   log  
2

From the data: a log   r  a  log    1 or log   1


2
log 
As  is acute, we take by  =1    e NOTE
Substituting   e in (iii), we get
tan1   2  
2e log e  2e 
 2 
 log e
 1
1  e log e  1 e 
2 e

 2e 
 1   2  tan 1  2 
1 e 

5. Consider Consider
a a
r as r
1 1 2
1 1 1 1
r

a

a 
1    1   2   a
r
Diff w.r.t  Diff w.r.t 
 2
1 dr 1 1
r d a
 
2
  2   a
dr
r d 2

1 dr r  2r 1 dr
 
r d a 2 a r d
d a 2 d  a
r  i.e r 
dr r dr 2r

a 2 a 1 2 
tan 1  tan 2    
a 2  a 
1   
 tan1   1    tan 2  
1
2
1 2 
Now, we have
a
 a 1   2   a 1   
a
r
1 1 2

or    3  1     3  1 or   1
 tan1  2 & tan 2   1

DEPT OF MATHS, SJBIT Page 26


ENGINEERING MATHEMATICS-I 15MAT11

tan 1  tan 2
Consider tan 1  2  
1  tan 1 tan 2 
2   1
  3  3
1  2  1
 1  2  tan 1 3

Pedal equations (p-r equations):- Any equation containing only p & r is


known as pedal equation of a polar curve.

Working rules to find pedal equations:-

(i) Eliminate r and  from the Eqs.: (i) r  f   & p  r sin 


2
1  dr 
(ii) Eliminate only  from the Eqs.: (i) r  f   &  2  2  4 
1 1

p r r  d 
 Find the pedal equations for the polar curves:-
2a
1.  1  cos 
r
2. r  e c cot 
3. r m  a m sin m  b m cos m (VTU-Jan-2005)
4. l  1  e cos 
r
Sol:
2a
1. Consider  1  cos  ……….(i)
r
Diff. w.r.t 
 
2a  1 2
r d
dr
 sin 
1 dr  r sin 

r d 2a
d 2a 1
r 
dr r sin 
1  cos   2 sin 2 
tan  
sin 

2 sin 
2
cos 
 
  tan 
2
2 2
 
tan  tan       
2 2
Using the value of  is p  r sin  , we get
 
p  r sin    r sin .............(ii )
2 2
Eliminating “  ” between (i) and (ii)

DEPT OF MATHS, SJBIT Page 27


ENGINEERING MATHEMATICS-I 15MAT11

 1  cos   r  2a 
2
p 2  r 2 sin 2   r2      [See eg: - (i)]
2  2  2 r 
p 2  ar.

This eqn. is only in terms of p and r and hence it is the pedal equation of the polar
curve.

2. Consider r  e cot 
Diff. w.r.t 
dr
 e cot  cot    r cot   r  e cot 

d
We use the equation
2
1 1 1  dr 
 2  4 
p 2
r r  d 

 2  4 r cot  
1 1 2

r r
 2  4 cot 2    2 1  cot 2    2 cos ec 2
1 1 1 1
r r r r
1 1
2
 2 cos ec 2
p r
2
p2  r or r 2  p 2 cos ec 2 is the required pedal equation
cos ec 2

3.Consider r  a sin m  b cos m


m m m

Diff. w.r.t 
dr
mrm 1  a m m cos m   bm ( m sin m )
d
m
r dr
 a m cos m  b m sin m
r d
1 dr a m cos m  b m sin m
 m
r d a sin m  b m cos m

a m cos m  b m sin m
cot  
a m sin m  b m cos m
1 1
Consider p  r sin  ,  cos ec
p r
1 1
2
 2 cos ec 2
p r

 2 1  cot 2  
1
r

DEPT OF MATHS, SJBIT Page 28


ENGINEERING MATHEMATICS-I 15MAT11

1   a m cos m  bm sin m  
2

 2 1   m  
r   a sin m  bm cos m  
 
1  a m sin m  bm cos m   a m cos m  bm sin m  
2 2

 2 
r 
 a sin m  b cos m 
m m 2


1 1  a 2m  b2m 
  
p2 r 2  r 2m 
r 2  m 1 
 p  2m
2
is the required p-r equation
a  b2 m

4. Consider l  1  cos  
r
Diff w.r.t 
 1 dr   dr 
l  2   e sin   l r  1 r   e sin 
 r d   d 
l cot    e sin 
r
 
 cot   r e sin 
l
We have 2  2 1  cot 2   (see eg: 3 above)
1 1
p r
1 1  l 2  e2 r 2 sin 2  

p 2 r 2  
Now
l2 

1
r

2
2 2
1  e r 2 sin 2 
l

lr
1  e cos   l  e cos  
r r
l r
2
l r 
cos      sin   1  cos    1   
2 2

 re   re 
 2   l  r  2  
2 2
 l  e r 1    
1 1   re   

p2 r2  l2 
 
 
 

1  e2  1  2
On simplification  
p 2  e 2  lr

DEPT OF MATHS, SJBIT Page 29


ENGINEERING MATHEMATICS-I 15MAT11

DERIVATIVES OF ARC LENGTH:


Consider a curve C in the XY plane. Let A be a fixed point on it. Let P and Q be
two neighboring positions of a variable point on the curve C. If „s‟ is the distance of P
from A measured along the curve then „s‟ is called the arc length of P. Let the tangent to
C at P make an angle  with X-axis. Then (s,) are called the intrinsic co-ordinates of
the point P. Let the arc length AQ be s + s. Then the distance between P and Q
measured along the curve C is s. If the actual distance between P and Q is C. Then
s=C in the limit Q  P along C.

y
Q

s
s
P(s, )
A


x
0

s
i.e. Lt 1
QP  C

Cartesian Form:

Q( x   x, y   y)

C s

P( x, y)

DEPT OF MATHS, SJBIT Page 30


ENGINEERING MATHEMATICS-I 15MAT11

Let y  f ( x) be the Cartesian equation of the curve C and let


P( x, y) and Q( x   x, y   y) be any two neighboring points on it as in fig.
Let the arc length PQ   s and the chord length PQ   C . Using distance between two

points formula we have PQ2 = ( C)2 =( x)2 +( y)2

 C   y  C  y 
2 2 2
   1   or  1  
 x   x  x  x 
2
 s  s C  s  y 
    1  
 x C  x C x 
s
We note that x  0 as Q  P along C, also that when Q  P, 1
C
 When Q  P i.e. when x  0, from (1) we get
2
ds  dy 
 1    (1)
dx  dx 
Similarly we may also write
2
s s C s  x 
   1   
y C y C  y 
and hence when Q  P this leads to
2
ds  dx 
 1    (2)
dy  dy 
Parametric Form: Suppose x  x(t ) and y  y(t ) is the parametric form of the curve C.
Then from (1)
2
 dy  2 2
ds   1  dx   dy 
 1   dt       
dx  dx dt  dx  dt   dt 
  dt

2 2
ds ds dx  dx   dy 
         (3)
dt dx dt  dt   dt 

Note: Since  is the angle between the tangent at P and the X-axis,

DEPT OF MATHS, SJBIT Page 31


ENGINEERING MATHEMATICS-I 15MAT11

dy
we have  tan
dx
ds
 1   y   1  tan 2   sec
2

dx
Similarly

ds 1 1
 1  1  1  cot 2   co sec
dy  y  2
tan 
2

dx dy
i.e. cos   and sin  
ds ds
2 2
 dx   dy 
       1   ds    dx    dy 
2 2 2
 ds   ds 
We can use the following figure to observe the above geometrical connections
among dx, dy, ds and  .

dy
ds


dx

DEPT OF MATHS, SJBIT Page 32


ENGINEERING MATHEMATICS-I 15MAT11

Polar Curves:
Suppose r  f ( ) is the polar equation of the curve C and P(r, ) and Q(r   r,   )
be two neighboring points on it as in figure:

C
Q(r   r,   )
N
s

P(r,)

 r


O x

Consider PN  OQ.
PN PN
In the right-angled triangle OPN, We have sin     PN  r sin   r
OP r
since Sin = when  is very small.
ON ON
From the figure we see that, cos     ON  r cos   r (1)  r
OP r
 cos   1 when   0
 NQ  OQ  ON  (r   r )  r   r

From  PNQ, PQ 2
 PN 2  NQ2 i.e, ( C )2  (r )2  ( r )2

C r   S  S C  S 2  r 
2 2

  r2        r  
      C   C   
S
We note that when Q  P along the curve,   0 also 1
C
2
dS  dr 
 when Q  P,  r2     (4)
d  d 

C   
2

Similarly, ( C )2  (r ) 2  ( r ) 2   1 r2  
r r 

DEPT OF MATHS, SJBIT Page 33


ENGINEERING MATHEMATICS-I 15MAT11

 S  S C  S   
2

and    1 r2  
r C r C r 

 d 
2
dS
 when Q  P, we get  1 r 2    (5)
dr  dr 
Note:
d
We know that tan   r
dr
2
ds  dr 
  r2     r  r cot   r 1  cot   rco sec 
2 2 2 2

d  d 
Similarly

 d 
2
ds
 1 r2    1  tan   sec
2

dr  dr 
dr d 1
  cos and  sin
ds ds r

The following figure shows the geometrical connections among ds, dr, d and 

ds r d


dr

Thus we have :
2
 dx 
2 2 2
ds  dy  ds ds  dx   dy 
 1   ,  1   ,     
dx  dx  dy  dy  dt  dt   dt 

DEPT OF MATHS, SJBIT Page 34


ENGINEERING MATHEMATICS-I 15MAT11

 d 
2 2
ds ds  dr 
 1 r2   and  r2   
dr  dr  d  d 
ds ds
Example 1: and for the curve x 2/3  y 2/3  a 2/3
dx dy
-1 1
2 2 x3  y 3
x 2/3  y 2/3  a 2/3  x -1/3  y -1/3 y '  0  y '  -1    
3 3 x
y3
2
ds  dy  y 2/3
Hence  1     1  2/3
dx  dx  x
x 2/3  y 2/3
1/3
a 2/3  a 
   
x 2/3 x 2/3  x 
Similarly
2
ds  dx  x 2/3 x 2/3  y 2/3
 1     1  2/3 
dy  dy  y y 2/3
1/3
a 2/3  a 
  
y 2/3  y 

ds  a2 
Example 2: Find for the curve y  a log  2 2 
dx  a -x 
 2 x 
y  a log a 2  a log  a 2  x 2  
dy 2ax
 a  2 2 
 2
a x  a x
2
dx
2
ds  dy  4a 2 x 2
  1    1 2
dx  dx   a  x2 
a  x 2   4a 2 x 2 a  x2 
2 2 2

 
a  x2  a  x2 
2 2 2 2

a2  x2

a2  x2

Example 3: If x  aet sint, y  aet cost, find ds


dt
dx
x  ae t sint   ae t sint  ae t cost
dt

DEPT OF MATHS, SJBIT Page 35


ENGINEERING MATHEMATICS-I 15MAT11

dy
y  aet cost   aet cost  aet sint
dt
2 2
ds  dx   dy 
        a 2e2t  cos t  sin t   a 2e2t  cos t  sin t 
2 2

dt  dt   dt 

 aet 2  cos 2t  sin2t   a 2 et   a  b    a  b   2  a 2  b2 


2 2

 t ds
Example 4: If x  a cos t  log tan  , y  a sin t, find
 2 dt

 
dx  sec 2 t   1 
 a   sin t  2   a  sin t 

dt  2  tan t  t t
 2  2 sin cos 
 2 2


 a   sin t 
1 
a
1  sin2t  a cos 2t
  a cost  cot t
 sin t  sin t sin t

dy
 a cos t
dt
2 2
ds  dx   dy 
     
dt  dt   dt 
 a 2 cos 2t cot 2t  a 2cos 2t

 a 2cos 2t  cot 2t  1

 a 2cos 2t  co sec2 t  a 2cot 2t


 a cot t

ds
Example 5: If x  a cos3 t, y  sin 3 t, find
dt
dx dy
 3 a cos 2t sin t ,  3 a sin 2t cos t
dt dt
2 2
ds  dx   dy 
       9a 2cos 4t sin2t  9a 2 sin4t cos 2t
dt  dt   dt 

 9a 2cos 2t sin2t  cos 2t  sin2t   3 a sin t cos t

DEPT OF MATHS, SJBIT Page 36


ENGINEERING MATHEMATICS-I 15MAT11

ds
Example 6: If r 2  a 2 cos 2 , Show that r is constant
d
dr dr a 2
r 2  a 2cos 2  2r  2a 2 sin2   sin2
d d r
2
ds  dr  a4 1 4
 r2     r 2
 sin2 2  r  a 4 sin 2 2
d  d  r 2
r

ds
r  r 4  a 4 sin2 2  a 4cos 2 2  a 4 sin2 2
d
ds
 a 2 cos 2  sin2 2  a 2  constant r is constant for r 2  a 2cos2
d

r k2  r2 ds
Example 7: For the curve   cos-1    , Show that r is constant.
k r dr

 2r 
  k  r (1)
2 2
r
d 1 1  2 k r 
2 2
1 r2  k 2  r2 
    
dr r2 k r2 k2  r2 r2 k2  r2
1 2
k

1 k2 r 2  k 2 k2  r2
   
k2  r2 r2 k2  r2 r2 k2  r2 r

ds  d 
  1 r2  2
dr  dr 

 1 r 2 k 2
 r2 

r2  k2  r2 k

r2 r r
ds
Hence r  k (constant)
dr
ds r ds r 2
Example 8: For a polar curve r  f   show that  , 
dr r 2  p 2 d p

dr d 1
We know that cos   and  sin 
ds ds r

dr p2 r 2  p2
  cos  1  sin2  1  2   p  rsin
ds r r

DEPT OF MATHS, SJBIT Page 37


ENGINEERING MATHEMATICS-I 15MAT11

ds r
 
dr r  p2
2

ds r r r2
Also   
d sin  p p
r

CURVATURE:
Consider a curve C in XY-plane and let P, Q be any two neighboring points on it.
Let arc AP=s and arc PQ=s. Let the tangents drawn to the curve at P, Q respectively
make angles  and + with X-axis i.e., the angle between the tangents at P and Q is
. While moving from P to Q through a distance„s‟, the tangent has turned through the
angle „‟. This is called the bending of the arc PQ. Geometrically, a change in 

represents the bending of the curve C and the ratio represents the ratio of bending of
s
C between the point P & Q and the arc length between them.

y 
Q
s
P

 +

o x

d 
Rate of bending of Curve at P is  Lt
ds Q  P  s
This rate of bending is called the curvature of the curve C at the point P and is denoted by
d
 (kappa). Thus   We note that the curvature of a straight line is zero since there
ds
exist no bending i.e. =0, and that the curvature of a circle is a constant and it is not equal
to zero since a circle bends uniformly at every point on it

DEPT OF MATHS, SJBIT Page 38


ENGINEERING MATHEMATICS-I 15MAT11

1
If   0, then is called the radius of curvature and is denoted by  (rho - Greek letter).

1 ds
  
 d
Radius of curvature in Cartesian form :
Suppose y = f(x) is the Cartesian equation of the curve considered in figure.

y
c


x
0

d2 y d d ds
 1  tan2 
dy
we have y   tan   y  2  sec 2   
dx dx dx ds dx
2
ds  dy 
But we know that  1  
dx  dx 
3
  dy 2  2

1    
d 2 y   dy   d ds   dx  
2 2
 dy 
 2  1       1    
dx   dx   ds  dx  d d2y
dx 2
3

ds 1   y  
 2
 2

  
d y
This is the expression for radius of curvature in Cartesian form.

DEPT OF MATHS, SJBIT Page 39


ENGINEERING MATHEMATICS-I 15MAT11
3
  dx  2  2

1    
  dy  
NOTE: We note that when y‟=, we find  using the formula   
 d 2x 
 2
 dy 
Example 9: Find the radius of curvature of the curve x3+y3 = 2a3 at the point (a, a).
x2
x3  y3  2a3  3x2  3 y 2  y  0  y   hence at  a, a  , y  1
y2

 y 2  2 x   x 2  2 y  y   2a 3  2a 3  4
 y    4  , hence at  a , a  , y     4  
 y   a  a
3 3
1   y 2  2
1   12  2

    a a
i.e.,   2 2 
y 4 4 2
a
Example 10: Find the radius of curvature for x  y  a at the point where it meets

the line y=x.


a
On the line y  x, x  x  a i.e 2 x  a or x
4
a a
i.e., We need to find  at  , 
4 4

1 1 y a a
x y a  y  0 i.e y   , hence at  ,  , y  1
2 x 2 y x 4 4

 1 1 
 x 2 y  y  y 2 x 
Also, y    
 x 
 
 
 a 1 a 1 
  (1)  
 42 a 4
2
a 1 1
(  )
a a 
 at  ,  , y    4 4    2 2   (1)  4
4 4 a  a a a
 
 4  4 4
 

DEPT OF MATHS, SJBIT Page 40


ENGINEERING MATHEMATICS-I 15MAT11
3 3
1   y 2  2
1   12  2

      a a
 2 2
y 4 4 2
a
Example 11: Show that the radius of curvature for the curve y  4 Sin x - Sin 2x

at x   is 5 5
2 4
y  4 sin x - sin 2x  y  4 cos x  2 cos 2x

 when x   , y  4cos   2cos  0  2(1)  2


2 2

Also, y  -4 sin x  4 sin 2x and when x   , y  -4 sin   4 sin   4


2 2
3
1   y 2  2 3
1  22  2

     5 5
   
y  4 4

Example 12: Find the radius of curvature for xy2 = a 3 - x3 at (a, 0).

xy 2  a3  x3  y 2  2 xy y  3x 2

3x 2  y 2
 y  and at (a, 0), y  
2 xy
dx 2 xy dx
In such cases we write  and at (a, 0), 0
dy 3x  y
2 2
dy

 2 2  dx   dx 
  3x  y   2 y  2 x   2 xy  6 x  2 y  
dx 2 xy d x  dy
2
  dy 
Also  2  2 
dy 3x  y  3x  y  
2 2
dy 2 2
 
 

d 2 x   3a  0   0  2a   0  6a3 2
 2 
 At  a, 0  ,   
dy 2    
2 4

 3a 2
 0 
9 a 3a

3
  dx 2  2

1     3
  dy   1  o 2  2

   2  3a
 or  
d x 2 2 2
dy 3a

DEPT OF MATHS, SJBIT Page 41


ENGINEERING MATHEMATICS-I 15MAT11

1. Find the radius of curvature of the curve


x  a log(sect  tant), y  asect
 x  a log(sect  tant)
dx a a sec t (seet  tant)
 sec t tan t  sec 2 t 
dt sec t  tan t (seet  tant)
dx
  a sec t
dt
Also y  a sec t gives
dy
 a sec t tan t
dt
dy dy dt a sec t tan t
Now , y1   
dx dt dx a sec t
y1  tan t
Differentiating w.r.t x we get
dt
y2  sec 2 t
dx
sec t
 y2 
a

1  y 
3
2 2

we have  
1

y2

 
3
a 1  tan 2 t 2


sec t
  a sec 2 t

DEPT OF MATHS, SJBIT Page 42


ENGINEERING MATHEMATICS-I 15MAT11

2.

DEPT OF MATHS, SJBIT Page 43


ENGINEERING MATHEMATICS-I 15MAT11

DEPT OF MATHS, SJBIT Page 44


ENGINEERING MATHEMATICS-I 15MAT11

DEPT OF MATHS, SJBIT Page 45


ENGINEERING MATHEMATICS-I 15MAT11

1.

DEPT OF MATHS, SJBIT Page 46


ENGINEERING MATHEMATICS-I 15MAT11

2.

3.

DEPT OF MATHS, SJBIT Page 47


ENGINEERING MATHEMATICS-I 15MAT11

DEPT OF MATHS, SJBIT Page 48


ENGINEERING MATHEMATICS-I 15MAT11

MODULE II

DIFFERENTIAL CALCULUS-II
CONTENTS:

 Taylor‟s and Maclaurin‟s theorems for function of o ne


variable……………………………………………………50
 Indeterminate forms……………………………………………… 53

 L‟Hospital‟s rule (without proof)…………………………………55

 Partial derivatives…………………………………………68

 Total derivative and chain rule…………………………72

 Jacobians –Direct evaluation……………………….75

DEPT OF MATHS, SJBIT Page 49


ENGINEERING MATHEMATICS-I 15MAT11

Taylor‟s Mean Value Theorem:

(Generalized Mean Value Theorem):

(English Mathematician Brook Taylor 1685-1731)

Statement:

Suppose a function f (x) satisfies the following two conditions:

(i) f (x) and it‟s first (n-1) derivatives are continuous in a closed interval a, b
(ii) f (n 1) ( x) is differentiable in the open interval a, b 
Then there exists at least one point c in the open interval a, b  such that

(b  a)2 (b  a)3
f (b)  f (a)  (b  a) f (a)  f (a)  f (a)  …..
2 3
(b  a)n 1 (n 1) (b  a)n ( n)
.....  f (a)  f (c)  (1)
n 1 n
Taking b  a  h and for 0    1 , the above expression (1) can be rewritten as
h2 h3 hn 1 (n 1) h n ( n)
f (a  h)  f (a)  hf (a) f (a)  f (a)  ....  f (a)  f (a   h)  (2)
2 3 n 1 n
Taking b=x in (1) we may write

( x  a)2 ( x  a)3 ( x  a)n 1 (n 1)


f ( x)  f (a)  ( x  a) f (a)  f (a)  f (a)  ...  f (a)  Rn  (3)
2 3 n 1
( x  a )n ( n)
Where Rn  f (c)  Re mainder term after n terms
n

When n  , we can show that Rn  0 , thus we can write the Taylor‟s series as

( x  a)2 ( x  a )n 1 ( n 1)
f ( x)  f (a)  ( x  a) f (a)  f (a )  ...  f (a)  ....
2 n 1

( x  a)n ( n)
 f (a)   f (a)  (4)
n 1
n

Using (4) we can write a Taylor‟s series expansion for the given function f(x) in powers
of (x-a) or about the point „a‟.

DEPT OF MATHS, SJBIT Page 50


ENGINEERING MATHEMATICS-I 15MAT11

Maclaurin‟s series:
(Scottish Mathematician Colin Maclaurin‟s 1698-1746)

When a=0, expression (4) reduces to a Maclaurin‟s expansion given by


x2 x n 1 ( n 1)
f ( x)  f (0)  xf (0)  f (0)  ...  f (0)  ....
2 n 1

x n ( n)
 f (0)   f (0)  (5)
n 1
n

Example 1: Obtain a Taylor‟s expansion for f ( x)  sin x in the ascending powers of


 
 x   up to the fourth degree term.
 4

The Taylor‟s expansion for f (x) about is
4
  
( x  )2 ( x  )3 ( x  )4
     4 f (4) (  ) ....  (1)
f ( x)  f ( )  ( x  ) f ( )  4 f ( )  4 f ( ) 
4 4 4 2 4 3 4 4 4

   1    1
f ( x)  sin x  f    sin  ; f ( x)  cos x  f     cos 
4 4 2 4 4 2
   1
f ( x)   sin x  f     sin  
4 4 2

   1
f ( x)   cos x  f     cos  
4 4 2

   1
f (4) ( x)  sin x  f (4)    sin 
4 4 2
Substituting these in (1) we obtain the required Taylor‟s series in the form
  
( x  )2 ( x  )3 ( x  )4
1  1 4 1 4 1 4 1
f ( x)   ( x  )( )  ( ) ( ) ( ) ....
2 4 2 2 2 3 2 4 2

  2   
( x  )3 ( x  ) 4
1   (x  4 ) 4 4 
f ( x)  1  ( x  )     ...
2 4 2 3 4 
 

Example 2………………………….: Obtain a Taylor‟s expansion for f ( x)  loge x up


to the term containing  x  1 and hence find loge(1.1).
4

DEPT OF MATHS, SJBIT Page 51


ENGINEERING MATHEMATICS-I 15MAT11

The Taylor‟s series for f (x) about the point 1 is


( x  1)2 ( x  1)3 ( x  1) 4 (4)
f ( x)  f (1)  ( x  1) f (1)  f (1)  f (1)  f (1) ....  (1)
2 3 4

1
Here f ( x)  loge x  f (1)  log 1  0 ; f ( x)   f (1)  1
x
1 2
f ( x)    f (1)  1 ; f ( x)   f (1)  2
2
x x3
6
f (4) ( x)    f (4) (1)  6 etc.,
4
x
Using all these values in (1) we get
( x  1)2 ( x  1)3 ( x  1)4
f ( x)  log e x  0  ( x  1)(1)  (1)  (2)  ( 6) ....
2 3 4

( x  1)2 ( x  1)3 ( x  1)4


 loge x  ( x  1)    ....
2 3 4

Taking x=1.1 in the above expansion we get

(0.1)2 (0.1)3 (0.1) 4


 log e (1.1)  (0.1)    ....  0.0953
2 3 4

Example 18: Using Taylor‟s theorem Show that


x 2 x3
loge (1  x)  x   for 0    1, x  0
2 3

Taking n=3 in the statement of Taylor‟s theorem, we can write


x2 x3
f (a  x)  f (a)  xf (a)  f (a)  f (a   x)  (1)
2 3
1 1 2
Consider f ( x)  loge x  f ( x)  ; f ( x)   and f ( x) 
2
x x x3
Using these in (1), we can write,
1 x  1  x  
2 3
2
log(a  x)  log a  x           (2)
 a  2  a 2  3  (a   x)3 
For a=1 in (2) we write,
x 2 x3 1 x 2 x3 1
log(1  x)  log1  x    x 
2 3 (1   x) 3 2 3 (1   x)3

DEPT OF MATHS, SJBIT Page 52


ENGINEERING MATHEMATICS-I 15MAT11

1
Since x  0 and   0, (1   x)3  1 and therefore 1
(1   x)3
x 2 x3
 log(1  x)  x  
2 3

Example 19: Obtain a Maclaurin‟s series for f ( x)  sin x up to the term containing x5 .

The Maclaurin‟s series for f(x) is

x2 x3 x 4 (4) x5 (5)
f ( x)  f (0)  x f (0)  f (0)  f (0)  f (0)  f (0) ....  (1)
2 3 4 5

Here f ( x)  sin x  f (0)  sin 0  0 f ( x)  cos x  f (0)  cos 0  1

f ( x)   sin x  f (0)  sin 0  0 f ( x)   cos x  f (0)  cos 0  1

f (4) ( x)  sin x  f (4) (0)  sin 0  0 f (5) ( x)  cos x  f (5) (0)  cos 0  1

Substituting these values in (1), we get the Maclaurin‟s series for f ( x)  sin x as
x2 x3 x4 x5
f ( x)  sin x  0  x (1)  (0)  (1)  (0)  (1) ....
2 3 4 5
x3 x5
 sin x  x   ....
3 5

Indeterminate Forms:

While evaluating certain limits, we come across expressions of the form


0 
, , 0  ,   , 00 , 0 and 1 which do not represent any value. Such expressions are
0 
called Indeterminate Forms.

We can evaluate such limits that lead to indeterminate forms by using L‟Hospital‟s Rule
(French Mathematician 1661-1704).

L‟Hospital‟s Rule:
If f(x) and g(x) are two functions such that

(i) lim f ( x)  0 and lim g ( x)  0


x a x a

DEPT OF MATHS, SJBIT Page 53


ENGINEERING MATHEMATICS-I 15MAT11

(ii) f ( x) andg ( x) exist and g (a)  0

f ( x) f ( x)
Then lim  lim
x a g ( x) x a g ( x )

The above rule can be extended, i.e, if


f ( x) f ( x) f ( x)
f (a)  0 and g (a)  0 then lim  lim  lim  .....
x a g ( x ) x a g ( x) x a g ( x)

Note:

0 
1. We apply L‟Hospital‟s Rule only to evaluate the limits that in , forms. Here we
0 
f ( x)
differentiate the numerator and denominator separately to write and apply the
g ( x)
0 
limit to see whether it is a finite value. If it is still in or form we continue to
0 
f ( x)
differentiate the numerator and denominator and write further and apply the
g ( x)
limit to see whether it is a finite value. We can continue the above procedure till we
get a definite value of the limit.

2. To evaluate the indeterminate forms of the form 0 ,   , we rewrite the functions
0 
involved or take L.C.M. to arrange the expression in either or and then apply
0 
L‟Hospital‟s Rule.
3. To evaluate the limits of the form 00 , 0 and 1 i.e, where function to the power of
function exists, call such an expression as some constant, then take logarithm on both
0 
sides and rewrite the expressions to get or form and then apply the L‟Hospital‟s
0 
Rule.

4. We can use the values of the standard limits like


sin x tan x x x
lim  1; lim  1; lim  1; lim  1; lim cos x  1; etc
x 0 x x 0 x x 0 sin x x 0 tan x x 0

DEPT OF MATHS, SJBIT Page 54


ENGINEERING MATHEMATICS-I 15MAT11

Evaluate the following limits:


sin x  x
Example 1: Evaluate lim
x 0 tan 3 x

sin x  x  0  cos x  1  0   sin x 0


lim    lim 2  
 lim 2  
x 0 tan x  0 
3 x  0 3tan x sec x 0 
2 x  0 6 tan x sec x  6 tan x sec x 0 
4 3

 cos x 1
 lim 
x 0 6sec x  24 tan x sec x  18 tan x sec x  12 tan x sec x
6 2 2 4
4 4 2
6

Method 2:

sin x  x
sin x  x  0  x3  0   lim sin x  x 0   lim tan x  1
lim 
  x 0
lim 3    
x 0 tan 3 x
0  tan x   0  x0 x 0
3 x 0 x
 
 x 
cos x  1  0   sin x  0   cos x 1
 lim    lim   lim 
x 0 3 x  0  x 0 6 x  0  x 0 6
2
6

ax  bx
Example 2: Evaluate lim
x 0 x

ax  bx  0 
lim  
x 0 x 0
a x log a  b x log b
 lim
x 0 1
a
 log a  log b log
b

x sin x
Example 3: Evaluate lim
e  1
x 0 x 2

x sin x  0  sin x  x cos x  0  cos x  cos x  x sin x 1  1  0 2


lim 2  
 lim x  
 lim   1
x 0
 e  1  0 
x x  0 2  e  1 e  0 
x x  0 2 e x .e x  (e x  1)e x  2[1  0] 2

DEPT OF MATHS, SJBIT Page 55


ENGINEERING MATHEMATICS-I 15MAT11

x e x  log(1  x)
Example 4: Evaluate lim
x 0 x2
1
ex  x ex 
1 ex  ex  x ex 
1  x   1  1  0  1  3
2
x e x  log(1  x)  0  1  x  0   lim
lim    lim   x 0
x 0 x2  0  x 0 2x 0 2 2 2

cosh x  cos x
Example 5: Evaluate lim
x 0 x sin x
cosh x  cos x  0  sinh x  sin x  0  cosh x  cos x 11 2
lim    lim    lim   1
x 0 x sin x  0  x0 sin x  x cos x  0  x0 cos x  cos x  x sin x 1  1  0 2

cos x  log(1  x)  1  x
Example 6: Evaluate lim
x 0 sin 2 x

1 1
 sin x  1  cos x 
cos x  log(1  x)  1  x  0  1  x  0   lim (1  x)2 1  1
lim    lim   x 0  0
x 0 sin 2 x  0  x 0 sin 2 x 0 2 cos 2 x 2

xx  x
Example 7: Evaluate lim
x 1 x  1  log x

xx  x  0  x x (1  log x)  1  0  x x (1  log x) 2  x x 1 1  1
lim    lim 
  x1 lim  2
x 1 x  1  log x  0  x1 1
1  0  1 1
x x2

1
since y  x x  log y  x log x  y
y
 1  log x  y  y (1  log x)
d x
then ( x )  x x (1  log x)
dx

sec2 x  2 tan x
Example 8: Evaluate lim
x
 1  cos 4 x
4

sec x  2 tan x  0 
2
2sec2 x tan x  2sec2 x  0  2sec4 x  4sec2 x tan 2 x  4sec2 x tan x
lim    lim    lim
x
 1  cos 4 x  0  x  4 sin 4 x  0  x  16 cos 4 x
4 4 4

DEPT OF MATHS, SJBIT Page 56


ENGINEERING MATHEMATICS-I 15MAT11

 2  2  (1)  2
4 2 2
2 4 2
4 8 1
  
16 16 2

log(sin x. cos ec a)
Example 9: Evaluate lim
x a log(cos a.sec x)
 cos x cos ec a 
 sin x. cos ec a 
log(sin x. cos ec a )  0     lim cot x  cot 2 a
lim    lim
x a log(cos a.sec x)  0  xa  sec x tan x.cos a  xa tan x
 cos a.sec x 

e x  e x  2cos x
Example 10: Evaluate lim
x 0 x sin x
e x  e x  2cos x  0  e x  e x  2sin x  0  e x  e x  2cos x 11 2
lim    lim    lim  2
x 0 x sin x  0  x0 sin x  x cos x  0  x0 cos x  cos x  x sin x 1  1  0

x cos x  log(1  x)
Example 11: Evaluate lim
x 0 x2
1
cos x  x sin x 
x cos x  log(1  x)  0  1 x  0 
lim    lim  
x 0 x2  0  x 0 2x 0

1
 sin x  sin x  x cos x 
(1  x) 2 0  0  0  1 1
 lim  
x 0 2 2 2

log(1  x 2 )
Example 12: Evaluate lim
x 0 log cos x

 2 x 
log(1  x )  0 
2 1  x 2  2 x cos x  0  2 cos x  2 x sin x 20
lim    lim   lim    lim  2
x 0 log cos x
0 x  0   sin x  x  0 (1  x )sin x  0 
2 x  0 (1  x ) cos x  2 x sin x 1  0
2

 
 cos x 

tan x  sin x
Example 13: Evaluate lim
x 0 sin 3 x

tan x  sin x  0  sec2 x  cos x  0  2sec2 x tan x  sin x  0 


lim    lim    lim  
x 0 sin 3 x  0  x0 3sin 2 x cos x  0  x0 6sin x cos 2 x  3sin 3 x  0 

DEPT OF MATHS, SJBIT Page 57


ENGINEERING MATHEMATICS-I 15MAT11

4sec2 x tan 2 x  2sec4 x  cos x 0  2 1 3 1


 lim   
x 0 6cos x  12sin x cos x  9 sin x cos x
3 2 2
600 6 2

Method 2:
tan x  sin x
tan x  sin x x3 tan x  sin x 0  sin x
lim  lim  lim    lim 1
0
3 3 3
x 0 sin x x 0
 sin x  x 0 x x 0 x
 
 x 
sec2 x  cos x 0  2sec2 x tan x  sin x 0 
 lim 
  x 0lim  
x 0 3x 2 0 6x 0

4sec2 x tan 2 x  2sec4 x  cos x 0  2  1 3 1


 lim   
x 0 6 6 6 2

tan x  x
Example 14: Evaluate lim
x 0 x 2 tan x

tan x  x
tan x  x x3 tan x  x 0  tan x
lim 2  lim  lim    lim 1
x 0  tan x 
0
x 0 x tan x x 0 3 
x x 0 x
 
 x 
sec2 x  1 0  2sec2 x tan x 0  4sec2 x tan 2 x  2sec4 x 0  2 1
 lim    lim 
  x 0lim  
x 0 3 x 2  0  x 0 6x 0 6 6 3

eax  e ax
Example 15: Evaluate lim
x 0 log(1  bx )

eax  e ax  0  aeax  ae  ax
lim 
  x 0 lim
x 0 log(1  bx ) 0 b / (1  bx)
 
a  a 2a
 
b b

a x  1  x log a
Example 16: Evaluate lim
x 0 x2
a x  1  x log a  0  a x log a  log a  0  a x (log a) 2 1
lim    lim    lim  (log a)2
 0  x 0  0  x 0
x 0 2
x 2x 2 2

e x  log(e  ex)
Example 17: Evaluate lim
x 0 x2

DEPT OF MATHS, SJBIT Page 58


ENGINEERING MATHEMATICS-I 15MAT11

e x  log(e  ex) e x  log e(1  x) e x  log e  log(1  x)  0 


lim  lim  lim  
x 0 x2 x 0 x2 x 0 x2 0
1 1
ex  ex 
1  x  0   lim (1  x)2 1  1
 lim    1
x 0 2 x  0  x 0 2 2


Limits of the form   :

log(sin 2 x)
Example 18: Evaluate lim
x 0 log(sin x)

log(sin 2 x)    (2cos 2 x / sin 2 x) 2cot 2 x 2 tan x  0  2sec 2 x 2


lim 
  x 0lim  lim  lim 
  x 0lim  1
x 0 log(sin x)
 (cos x / sin x) x 0 cot x
 
x 0 tan 2 x 0 2sec2 2 x 2

log x
Example 19: Evaluate lim
x 0 cos ecx

log x    1\ x  sin 2 x  0  2sin 2 x 0


lim    lim  lim    lim  0
x 0 cos ecx 
  x0  cos ec x.cot x x0 x cos x  0  x0 cos x  x sin x 1  0

log cos x
Example 20: Evaluate lim
 tan x
x
2

log cos x     tan x  tan x  sin x cos x 0


lim    lim   lim  lim  0
x
 tan x    x sec x 2 
x  sec x
2
x
 1 1
2 2 2 2

log(1  x)
Example 21: Evaluate lim
x 1 cot  x

log(1  x)    1/(1  x) sin 2  x  0  2 sin  x cos  x 0


lim    lim  lim    lim  0
x 1 cot  x
   x1  cos ec  x x1  (1  x)  0  x1  
2

Example 22: Evaluate lim log tan 2 x tan 3x


x 0

 log tan 3x     log e a


lim log tan 2 x tan 3x  lim     logb a 
x 0 
x 0 log tan 2 x
   log e b

DEPT OF MATHS, SJBIT Page 59


ENGINEERING MATHEMATICS-I 15MAT11

 3sec2 3x / tan 3x   3/ sin 3x.cos 3x   3/ sin 3x.cos 3x 


 lim    lim    lim  
 x0  2 / sin 2 x.cos 2 x  x0  2 / sin 2 x.cos 2 x 
2
x 0 2sec 2 x / tan 2 x

 6 / sin 6 x   6sin 4 x  0   24cos 4 x  24


 lim    lim     lim   1

x 0 4 / sin 4 x
 x0  4sin 6 x  0  x0  24cos 6 x  24

log( x  a)
Example 23: Evaluate lim
x a log(e x  ea )

log( x  a)    1/( x  a) (e x  e a )  0  ex ea
lim 
  x a x x a
lim  lim    lim  1
x a log(e x  e a )  e /(e  e ) xa e x ( x  a)  0  xa e x ( x  a)  e x e a
 

Limits of the form  0   : To evaluate the limits of the form  0   , we rewrite the
0 
given expression to obtain either   or   form and then apply the L‟Hospital‟s Rule.
0 
1
Example 24: Evaluate lim(a x  1) x
x 

 1 
1
1
a x (log a)  2 
1
(a  1)  0  x
x 
lim(a x  1) x  0   form   lim    lim
x  x   1 
0 x  
 1
   2
 x x 
1
 lim a x (log a)  a 0 log a  log a
x 

Example 25: Evaluate lim(1  sin x) tan x



x
2

(1  sin x)  0 
lim(1  sin x) tan x  0   form   lim  
  cot x  0 
x x
2 2

 cos x 0
 lim  0
x
  cos ec x 1
2
2


Example 26: Evaluate limsec .log x
x 1 2x
 log x  0 
.log x    0 form   lim
  
limsec
cos  0 
x 1 2x x 1

2x

DEPT OF MATHS, SJBIT Page 60


ENGINEERING MATHEMATICS-I 15MAT11

1/ x 2x 2
 lim  lim 
    1  x 1  
x
2   sin  2   sin
2 2 x  x  2x

Example 27: Evaluate lim x log tan x


x 0

log tan x   
lim x log tan x  0   form   lim  
x 0 x 0 (1/ x)   

sec2 x / tan x  x2
 lim  lim
x 0  1  x 0 sin x.cos x
 2
x 

2 x 2  0  4 x 0
 lim    lim  0
  x0 2cos 2 x 2
x 0 sin 2 x 0

x
Example 28: Evaluate lim (1  x 2 ) tan
x 1 2
x
lim (1  x 2 ) tan
x 1
 0   form 
2
1  x2  0  2 x
 lim    lim
x 1 x 0 x 1  x
cot  cos ec 2
2 2 2
2 4
 
  
 
2

Example 29: Evaluate lim tan x.log x


x 0

log x   
lim tan x.log x  0   form   lim  
x 0 x 0 cot x   
1/ x  sin 2 x  0   sin 2 x 0
 lim  lim    lim  0
x 0  cos ec 2 x x 0 x 0 x  0 1 1

Limits of the form      : To evaluate the limits of the form      , we take


0 
L.C.M. and rewrite the given expression to obtain either   or   form and then apply
0 
the L‟Hospital‟s Rule.

DEPT OF MATHS, SJBIT Page 61


ENGINEERING MATHEMATICS-I 15MAT11

1 
Example 30: Evaluate lim   cot x 

x 0 x

1   1 cos x 
lim   cot x   lim       form 

x 0 x
 x0  x sin x 

 sin x  x cos x   0   cos x  cos x  x sin x 


 lim      lim
x 0
 x sin x  0  x  0
 sin x  x cos x 

 x sin x  0 
 lim   
x  0 sin x  x cos x  0
  
 sin x  x cos x 
 lim 
x  0 cos x  cos x  x sin x 
 
00
 0
11 0

Example 31: Evaluate lim sec x  tan x 



x
2

 1 sin x 
lim sec x  tan x   lim       form 
x
  cos x
x  cos x 
2 2

1  sin x   0    cos x  0
 lim      lim   1 0
x    sin x 

x   cos x   0 
2 2

 1 x 
Example 32: Evaluate lim  

x 1 log x x  1

 1 x   ( x  1)  x log x   0 
lim        form   lim   

x 1 log x x  1 x 1
 ( x  1) log x   0 

     
 1  1  log x    log x   0    l/ x  1 1
 lim    lim  1     lim  1 1 
x 1 x  1 
  log x 
x 1
 1   log x   0  x 1
   1 1 2
 x   x  x 2
x

DEPT OF MATHS, SJBIT Page 62


ENGINEERING MATHEMATICS-I 15MAT11

1 1 
Example 33: Evaluate lim   x 

x 0 x e  1

1 1   (e x  1)  x   0 
lim   x      form   lim   

x 0 x e  1 x 0
 x(e  1)   0 
x

 ex 1 0  ex 
 lim  x     lim  
x 0 (e  1)  xe
  0  x0  e  e  xe 
x x x x

1 1
 
11 0 2

 1 1
Example 34: Evaluate lim   

x 0 sin x x

 1 1  x  sin x   0 
lim        form   lim   

x 0 sin x x x 0
 x sin x   0 

 1  cos x   0   sin x  0
 lim      lim    0
x 0 sin x  x cos x x 0 cos x  cos x  x sin x
  0   11

 1 log(1  x) 
Example 35: Evaluate lim   

x 0 x x2

 1 log(1  x)   x  log(1  x)   0 
lim     lim    0 
  x 0 
2
x 0 x x x2
 1   1 
 1  0  (1  x) 2  1
 lim  1  x     lim  
x 0
 2 x   0  x 0
 2  2
   
a x
Example 36: Evaluate lim   cot 
x 0 x
 a

 x  x
a x  a cos a   a cos a   0 
lim   cot   lim        form  xlim   x 

x 0 x a  x0  x sin x  0
 x sin   0 
 a  a

DEPT OF MATHS, SJBIT Page 63


ENGINEERING MATHEMATICS-I 15MAT11

 x x  1 x x x x
 a sin a  x cos a   0   a. a cos a  cos a  a sin a 
 lim     xlim  
 0
x 0 x 0 x x x
 x sin  sin  cos 
 a   a a a 

 x x   1 x x 1 x 
 sin   sin  . .cos 
a a  0  a a a a a 00
 lim      lim    0
 sin  cos   0 
x 0 x x x x 0 1 x 1 x x x 1 1
 cos  cos  2 sin   0
 a a a a a a a a a a a

sin 2 x  a sin x
Example 37: Find the value of „a‟ such that lim is finite. Also find the
x 0 x3
value of the limit.

sin 2 x  a sin x  0  2cos 2 x  a cos x 2  a


Let A= lim    lim   finite
0
3
x 0 x x  0 3x 2 0

We can continue to apply the L‟Hospital‟s Rule, if 2+a=0 i.e., a= -2 .

For a  2 ,
2cos 2 x  2cos x  0  4sin 2 x  2sin x  0 
A  lim    lim  
 0  x 0 0
x 0 2
3x 6x

8cos 2 x  2cos x 8  2
 lim   1
x 0 6 6

The given lim it will have a finite value when a  2 and it is 1.

x(1  a cos x)  b sin x 1


Example 38: Find the values of „a‟ and „b‟ such that lim  .
x 0 x3 3

x(1  a cos x)  b sin x  0  (1  a cos x)  ax sin x  b cos x 1  a  b


Let A  lim    lim   finite
 0  x 0
3
x 0 x 3x 2 0

We can continue to apply the L‟Hospital‟s Rule, if 1-a+b=0 i.e., a-b = 1 .

For a  b  1 ,

(1  a cos x)  ax sin x  b cos x  0  2a sin x  ax cos x  b sin x  0 


A  lim    lim  
0 0
x 0 2 
3x x 0 6x

DEPT OF MATHS, SJBIT Page 64


ENGINEERING MATHEMATICS-I 15MAT11

3a cos x  ax sin x  b cos x 3a  b


 lim   finite
x 0 6 6

1 3a  b 1
This finite value is given as . i.e.,   3a  b  2
3 6 3
1 1
Solving the equations a  b  1 and 3a  b  2we obtain a  and b   .
2 2

a cosh x  b cos x
Example 39: Find the values of „a‟ and „b‟ such that lim  1.
x 0 x2

a cosh x  b cos x a  b
Let A  lim   finite
x 0 x2 0
We can continue to apply the L‟Hospital‟s Rule, if a-b=0, since the denominator=0 .

For a  b  0 ,
a cosh x  b cos x  0 
A  lim  
x 0 x2 0
a sinh x  b sin x  0 
 lim  
x 0 2x 0
a cosh x  b cos x a  b
 lim 
x 0 2 2

But this is given as 1.


a  b  2

Solving the equations a  b  0 and a  b  2 we obtain a  1 and b  1.

Limits of the form 00 , 0 and 1 : To evaluate such limits, where function to the
power of function exists, we call such an expression as some constant, then take
0 
logarithm on both sides and rewrite the expressions to get or form and then apply
0 
the L‟Hospital‟s Rule.

Example 40: Evaluate lim x x


x 0

Let A  lim x x (00 form)


x 0
Take log on both sides to write

DEPT OF MATHS, SJBIT Page 65


ENGINEERING MATHEMATICS-I 15MAT11

log x   
log e A  lim log x x  lim x.log x (0   form)  lim  
x 0 x 0 x 0 1/ x


1/ x x 0
 lim  lim  0
x 0 (1/ x )
2 x  0 1 1

loge A  0  A  e0  1  lim x x  1
x 0

1
x2
Example 41: Evaluate lim(cos x)
x 0
1

Let A  lim(cos x) x2
(1 form)
x 0

Take log on both sides to write


1
1 log cos x  0 
log e A  lim log(cos x) x2
 lim log cos x ( 0 form)  lim  
x 0 x 0 x 2 x  0 x2  0 

 tan x  0   sec2 x 1
 lim    lim 
x 0 2 x  0  x 0 2 2

1 1
1 1 1
log e A    A  e 2   lim(cos x) x 
2

2 e x 0 e

Example 42: Evaluate lim(tan x)cos x



x
2

Let A  lim(tan x)cos x (0 form)



x
2

Take log on both sides to write

log e A  lim log(tan x)cos x  lim cos x log(tan x) (0   form)


 
x x
2 2

log tan x    sec2 x / tan x cos x 0


 lim    lim  lim 2   0
x
 sec x    x sec x.tan x x sin x 1
2 2 2

log e A  0  A  e0  1  lim(tan x)cos x  1



x
2

DEPT OF MATHS, SJBIT Page 66


ENGINEERING MATHEMATICS-I 15MAT11
1
tan x x2
Example 43: Evaluate lim( )
x 0 x
1
tan x x2 
Let A  lim( ) (1 form)
x 0 x

Take log on both sides to write


1
tan x x2 1 tan x
log e A  lim log( )  lim 2 log( ) (  0 form)
x 0 x x  0 x x
tan x sec 2 x 1
log( ) 
x 0 tan x x
 lim    lim
x 0 x2  
0 x 0 2 x
1 1 2 1
 
2 x  sin 2 x  0 
 lim sin x.cos x x  lim sin 2 x x  lim 2  
x 0 2x x  0 2x x  0 2 x sin 2 x  0 

2  2cos 2 x 0 4sin 2 x 0


 lim    lim  
x 0 4 x sin 2 x  4 x cos 2 x  0 
2 x  0 4sin 2 x  16 x cos 2 x  8 x sin 2 x  0 
2

8cos 2 x 8 1
 lim  
x 0 24cos 2 x  48 x sin 2 x  16 x cos 2 x
2
24 3

1 1 1
1  tan x x2 
log e A    A  e 3  lim( ) e 3
3 x 0 x
1
Example 44: Evaluate lim(a x  x) x
x 0
1
Let A  lim(a x  x) x (1 form)
x 0

Take log on both sides to write

1
1
log e A  lim log(a x  x) x  lim log(a x  x) (  0 form)
x 0 x 0 x

log(a  x)  0 
x
(a x log a  1) /(a x  x)
 lim 
  x 0lim
x 0 x 0 1

 log a  1  log a  log e  log ae

1
 log e A  log ea  A  ea Hence lim(a x  x) x  ea.
x 0

DEPT OF MATHS, SJBIT Page 67


ENGINEERING MATHEMATICS-I 15MAT11

x tan  x
Example 45: Evaluate lim(2  ) 2 a
x a a
x tan 2 ax 
Let A  lim(2  ) (1 form)
x a a

Take log on both sides to write


x tan 2 ax x x
log e A  lim log(2  )  lim tan .log(2  )   0 form 
x a a x  a 2a a

 (1/ a) 
x  x  x
log(2  )  2  sin 2
 lim a    lim 
0 a 2
  lim . 2a  2
 
 x  0  x a    x  x a 
x a
cot  cos ec 2 2
x 
2a  2a 2a  a
 
 

2 2
x tan 2 ax 2
 log e A   Ae 
Hence lim(2  ) e .

 x a a

PARTIAL DIFFERENTIATION:
Introduction: We often come across qualities which depend on two or more variables.
For e.g. the area of a rectangle of length x and breadth y is given by
Area = A(x,y) = xy. The area A(x, y) is, obliviously, a function of two variables.
Similarly, the distances of the point (x, y, z) from the origin in three-dimensional space is
an example of a function of three variables x, y, z.

Partial derivatives: Let z = f(x, y) be a function of two variables x and y.


z f
The first order partial derivative of z w.r.t. x, denoted by or or zx or fx is defined
x x
z f x  x, y   f x, y 
as  lim
x x  0 x
 z 
From the above definition, we understand that   is the ordinary derivative of z
 x 
w.r.t x, treating y as constant.
z f
The first order partial derivative of z w.r.t y, denoted by or or zy or fy is defined as
y y

DEPT OF MATHS, SJBIT Page 68


ENGINEERING MATHEMATICS-I 15MAT11

z f x, y  y   f x, y 
 lim
y y 0 y
 z 
From the above definition, we understand that   is the ordinary derivative of z
 y 
w.r.t y, treating x as constant
  z   2 z 2 f
The partial derivatives    2 or or zxx or fxx;
x  x  x x 2
  z   2 z 2 f
   2 or or zyy or fyy;
y  y  y y 2
  z   2 z
  or zyx or fyx
y  y  xy
  z   2 z
and   or zxy or fxy
y  y  xy
are known as second order Partial derivatives.

In all ordinary cases, it can be verified that


2z 2z

xy yx
The third and higher order partial derivatives of f(x,y) are defined in an analogous
way Also, the second and higher order partial derivatives of more than two
independent variables are defined similarly.

A note on rules of partial differentiation:-


All the rules of differentiation applicable to functions of a single
independent variable are applicable for partial differentiation also; the only
difference is that while differentiating partially w.r.t one independent variable all
other independent variables are treated as constants.

Total derivatives, Differentiation of Composite and Implicit functions

In this lesson we learn the concept of total derivatives of functions of two or


more variables and, also rules for differentiation of composite and implicit functions.

a) Total differential and Total derivative:-


For a function z  f ( x, y) of two variables, x and y the total differential (or
exact differential ) dz is defined by:
f f
dz  dx  dy --------------------------(1)
x x
Further, if z  f ( x, y) where x  x(t ) , y  y(t ) i.e. x and y are themselves
functions of an independent variable t, then total derivative of z is given by

DEPT OF MATHS, SJBIT Page 69


ENGINEERING MATHEMATICS-I 15MAT11

dz f dx f dy
  -------------------------(2)
dt x dt y dt
Similarly, the total differential of a function u  f ( x, y, z) is defined by
f f f
du  dx  dy  dz -----------------(3)
x y z
Further, if u  f ( x, y, z) and if x  x(t ) , y  y(t ) , z  z (t ) , then the total
derivative of u is given by

du f dx f dy f dz
   ---------------(4)
dt x dt y dt z dt

(b) Differentiation of implicit functions:-

An implicit function with x as an independent variable and y as the dependent


variable is generally of the form z  f ( x, y)  0 . This gives
 dz   df 
      0 .Then, by virtue of expression (2) above, we get
 dx   dx 

dz f f dy df f f dy
  or   , and hence
dx x y dx dx x y dx
 f 
f f dy x 
 
dy
0  , so that we get ----------- (5)
x y dx dx  f 
 y 

(c) Differentiation of composite functions:-


Let z be an function of x and y and that x   (u, v) and y   (u, v) are functions
of u and v then,

z f x f y
 
u x u y u

z f x f y
&   -------------------(6)
v x v y v

Similarly, if z  f (u, v) are functions of u and v and if u   ( x, y) and


v   ( x, y) are functions of x and y then,

DEPT OF MATHS, SJBIT Page 70


ENGINEERING MATHEMATICS-I 15MAT11

z f u f v
 
x u x v x

z f u f v
&   --------------------------(7)
y u y v y

Note:-1) The above formulae can be extended to functions of three are more
variables and formulas (6) and(7) are called Chain rule for partial differentiation.

2) The second and higher order partial derivatives of z  f ( x, y) can be obtained


by repeated applications of the above formulas

Evaluate:
1. Find the total differential of
(i) e x x sin y  y cos y  (ii) e xyz
Sol:- (i) Let z  f x, y   e x x sin y  y cos y Then
z
 e x 1  x sin y  y cos y 
x
z
and  e x 1  x  cos y  y sin y  Hence, using formula (1), we get
y
z z
dz  dx  dy
x y
i.e dz  e 1  x sin y  y cos ydx  e x 1  x cos y  y sin ydy
x

(ii) Let z  f ( x, y, z )  e xyz Then


u u u
  yz e xyz ;  xz e xyz ;  xy e xyz
x y z
 Total differential of z  f ( x, y, z ) is (see formula (3) above)
u u u
du  dx  dy  dz
x y z
 e xyz  yzdx  zxdy  xydz 
dz
2. Find if
dt
(i) z  xy 2  x 2 y ,where x  at 2 , y  2at
(ii) u  tan 1  y  ,where x  e t  e t , y  e t  e t (VTU-Jan 2003)
 x
Sol:- (i) Consider z  xy 2  x 2 y
z z
 y 2  2 xy &  2 xy  x 2
x y

DEPT OF MATHS, SJBIT Page 71


ENGINEERING MATHEMATICS-I 15MAT11

dx dy
Since x  at 2 & y  2at , We have  2at ,  2a
dt dt
Hence, using formula (2), we get
dz z dx z dy
 
dt x dt y dt
   
 y 2  2 xy 2at   2 xy  x 2 2a 
   
 y 2  2 xy y  2a 2 xy  x 2 , Using y  2at
dz
 y 3  2 xy 2  4axy  2ax 2
dt
 dz 
To get   explicitly in terms of t, we substitute
 dt 
x  at & y  2at , to get
2

 dz 
   2a 8t  5t
3 3
 4

 dt 

(ii) Consider
u  tan 1  y 
 x
u y u x
 2 ,  2
x x  y y x  y 2
2

Since x  e t  e t & y  e t  e t ,we have


dx dy
 e t  e t  y  e t  e t  x
dt dt
du u dx u dy
Hence   ( see eqn (2))
dt x dt y dt
 y   x   x2  y2 
  2 2 
 y   2  x   2
2 

2 
x y  x y  x y 
Substituting x  e t  e t & y  e t  e t , we get
du 2
 2t
dt e  e 2t
 dy 
3. Find   if (i) x y  y x =Constant
 dx 
(ii) x  e y  2 xy
Sol: - (i) Let z  f ( x, y)  x y  y x =Constant. Using formula (5)
f
  x -----------------(*)
dy
dx f
y

DEPT OF MATHS, SJBIT Page 72


ENGINEERING MATHEMATICS-I 15MAT11

f f
But  yx y 1  y x log y and  x y log x  xy x1 Putting those in(*), we get
x y
dy  yx y 1  y x log y 
  y x 1 
dx  x log x  xy 
(ii) Let z  f ( x, y)  e x  e y  2 xy =Constant
f f
Now,  ex  2y ;  e y  2 x Using this in (8),
x y
 f 
dy  x  e x  2 y 
     y 
dx  f   e  2x 
 y 

4. (i) If z  f ( x, y) ,where x  r cos  , y  r sin  show that


2
 z   z   z  1  z 
2 2 2

         2   (VTU July-2005)
 x   y   r  r   

(ii) If z  f ( x, y) ,where x  e u  e v & y  e u  e v ,Show that


z z z z
 x y
u v x y

Sol: As x  r cos  and y  r sin  , we have


x x y y
 cos  ,  r sin  ;  sin  &  r cos  .
r  r 
Using Chain rule (6) & (7) we have
 z  z x z y z z
    cos    sin  
 r  x r y r x y

 z  z x z y z z
     r sin    r cos  
   x  y  x y
Squaring on both sides, the
above equations, we get
2
 z   z   z   z  z 
2 2

     cos     sin   2   sin  cos 


2 2

 r   x   y   x  y 

2
1  z   z   z   z  z 
2 2

2      cos     sin   2   sin  cos 


2 2

r     x   y   x  y 

Adding the above equations , we get

DEPT OF MATHS, SJBIT Page 73


ENGINEERING MATHEMATICS-I 15MAT11

 z 
2
 z 
2 
 z   z  
2 2

   2
1 

         cos   sin 
2 2

 r  r    x
    
 y

2
 z   z 
2

=      as desired.
 x   y 
(ii) As x  e u  e v & y  e u  e v , We have
x x y y
 eu ,  e  v ,  e  u &  e v
u v u v
Using Chain rule (6) we get
 z  z x z y z u z v
    e   e 
 u  x u y u x y
 z  z x z y z z
     e v    e v 
 v  x v y v x y

 z   z  z u z u
    
 



e  e v  

e  ev
   
u v x y
z z
 x y
x y
u u u
5. (i) If u  f ( x, z, y / z) Then show that x y z 0 (VTU-July-2004)
x y z
(ii) If H  f ( x  y, y  z, z  x) , show that
H H H
  0 (VTU-July-2003)
x y z
Sol: - (i) Let u  f (v, w) , where v  xz and w  y
z
v v v w w 1 w  y
z,  0, x &  0,  , 
x y z x y z z z2
Using Chain rule,
u u v u w u
   z   u 0  z u
x v x w x v w v

u u v u w u
 
y v y w y v

w
 
0  u 1 z  1 u
z w
u u v u w u
   x   u   y z 2   x u  y2 u
z v z w z v w   v z w
From these, we get
u u u u y u  u y u 
x y z  xz   z x  
x y z v z w  v z 2 w 
=0
(ii) Let H  f (u, v, w) Where u  x  y, v  y  z, w  z  x

DEPT OF MATHS, SJBIT Page 74


ENGINEERING MATHEMATICS-I 15MAT11

u u u
Now,  1,  1, 0
x y z
v v v
 0,  1,  1
x y z
w w u
 1,  0,  1 Using Chain rule,
x y z
H H u H v H w H
    1  H 0  H  1
x u x v x w x u v w
H H u H v H w H
     1  1  v 0
H
y u y v y w y u v w
H H u H v H w H
    0  H  1  H 1
z u z v z w z u v w
u u u
Adding the above equations, we get the required result x y z 0
x y z

Applications to Jacobians:
In this lesson, we study Jacobians, errors and approximations using the
concept of partial differentiation.

Jacobians:-
Jacobians were invented by German mathematician C.G. Jacob Jacobi (1804-
1851),who made significant contributions to mechanics, Partial differential equations and
calculus of variations.

Definition:- Let u and v are functions of x and y, then Jacobian of u and v w.r.t x and
denoted by
 u, v   u, v 
J or J   or
 x, y    x , y 
is defined by
u u
 u, v  x y
J   
 x, y  v v
x y
Similarly, if u, v, w are functions of three independent variables of x, y, z, then

DEPT OF MATHS, SJBIT Page 75


ENGINEERING MATHEMATICS-I 15MAT11

u u u
x y z
 u, v, w  v v u
J  J   
 x, y, z  x y z
w w w
x y z
Remark:- In a similar way, Jacobian of n functions in n-variables can be defined
u, v 
Note:- (i) If J  , then the "inverse Jacobian" of the Jacobian J,
  x, y 
denoted by J  ,is defined as

x, y 
J 
u, v 
u, v, w   x, y , z 
(ii) Similarly, "inverse Jacobian" of J  is defined as J  
  x, y , z  u, v, w

Properties of Jacobians :-

u, v    x, y 
Property 1:- If J  and J   then JJ   1
  x, y  u, v 

Proof:- Consider
u u x x
u, v   x, y  x y u v
JJ 1    
 x, y   u, v  v v y y
x y u v
u x u y u x u y
 
x u y u x v y v 1 0
  1
v x v y v x v y 1 0
 
x u y u x v y v

Property 2:- (Chain rule for Jacobians):- If u and v are functions of r&s and r,s
are functions x&y,then
 u, v   u, v   r , s 
J     J    J  
 x, y   r , s   x, y 

DEPT OF MATHS, SJBIT Page 76


ENGINEERING MATHEMATICS-I 15MAT11

Proof:- Consider
u u r r
 u, v   r , s  r s x y
J   J    
 r , s   x, y  v v s s
r s x y
u r u s u r u s
 
r x s x r y s y

v r v s v r v s
 
r x s x r y s y
u u
x y  u, v 
  J  
v v  x, y 
x y

Jacobians in various co-ordinate systems:-

1. In Polar co-ordinates, x  r cos  , y  r sin 


u, v 
we have r
r , 
2. In spherical coordinates, x   cos  , y   sin  , z  z, we have
  x, y , z 

  ,  , z 

3. In spherical polar co-ordinates, x  r sin  cos  , y  r sin  sin  , z  r cos 


x y
Proof of 1:- we have,  cos  and  sin 
r r
x y
 r sin  and  r cos 
 

x x
 x, y  r  cos   r sin 
  
 r ,  y y sin  r cos 
r 
 
 r cos 2   r sin 2   r cos 2   sin 2   r

x y z
Proof of 2 :-we have  cos  ,  sin  , 0
  
x y z
   sin  ,   cos  , 0
  

DEPT OF MATHS, SJBIT Page 77


ENGINEERING MATHEMATICS-I 15MAT11

z z z
 0,  0,  0
  z
x x x
  z
cos    sin  0
 x, y, z  y y y
   sin   cos  0 
  ,  , z    z
0 0 1
z z z
  z

Proof of 3:- We have


x x x
 sin  cos  ,  r cos  cos  ,  r sin  sin 
r  
y y y
 sin  sin  ,  r cos  sin  ,  r sin  cos 
r  
z z z
 cos  ,  r sin  , 0
r  
sin  cos  r cos  cos   r sin  sin 
  x, y , z 
  sin  sin  r cos  sin  r sin  cos 
r , ,  
cos   r sin  0
 r 2 sin 

Evaluate
1. If u  x 2  2y 2 , v  2 x 2  y 2 , where
x  r cos  , y  r sin  show that
u, v 
 6r 3 sin 2 (VTU-Jan-2006)
r , 
Consider u  x 2  2 y 2  r 2 cos 2   2r 2 sin 2 
v  2 x 2  y 2  2r 2 cos 2   r 2 sin 2 
u v
  2r cos 2   4r sin 2  ,  4r cos 2   2r sin 2 
r r
u
 2r 2 cos  sin   4r 2 sin  cos 

v
 4r 2 cos  sin   2r 2 sin  cos 

u u
u, v  r  2r cos 2   4r sin 2   2r 2 cos  sin   4r 2 sin  cos 
 
r ,  v v 4r cos 2   2r sin 2   4r 2 cos  sin   2r 2 sin  cos 
r 

DEPT OF MATHS, SJBIT Page 78


ENGINEERING MATHEMATICS-I 15MAT11

 
 2r cos 2   4r sin 2   4r 2 cos  sin   2r 2 sin  cos  
  2r cos  sin   4r sin  cos  4r cos
2 2
  2r sin  
2

 6r sin 2
3

 x, y  1  x, y 
2. If x  u1  v , y  uv, Prove that J   J   1 (VTU-2001)
 u, v   u, v 
x x
Consider  1 v ,  u
u v
y y
 v, u
u v
x x
 x, y  u v 1  v  u
J   
 u, v  y y v u
u v
 1  v u   uv   u  uv  uv  u
 x, y 
J    u    (1)
 u, v 
Further, as x  u1  v , y  uv,
 u  uv
We write, x  u  y u  x  y and
y  y   y 
v     v   
u x y x y
u u
  1,  1 and
x y
v y v x
 , 
x x  y  y x  y 2
2

u u
1 1
1  u, v  x y
J     y x
 x, y  v v 
x  y  x  y 2
2

x y
x y  1  1 1
       JJ 1  u 1
x  y 2
x  y 2
x y u u

3. If x  e u cos v, y  e u sin v, Prove that


x, y  u, v 
 1
u, v  x, y 
Consider x  e u cos v y  e u sin v

DEPT OF MATHS, SJBIT Page 79


ENGINEERING MATHEMATICS-I 15MAT11

x y
 e u cos v  e u sin v
u u
x y
 e u sin v  e u cos v
v v
x x
 x, y  u v e cos v  e sin v
u u

  
 u, v  y y eu sin v eu cos v
u v
x, y 
i.e  e 2u      (1)
u, v 
Again Consider x  e u cos v, y  e u sin v,
1
 x 2  y 2  e2u or u  log x 2  y 2
2
 
&  tan v or v  tan 1  y 
y
x  x
u x u y
Hence  2 ,;  2 ,;
x x  y 2
y x  y 2
v y v x
&  2 ;  2
x x  y y x  y 2
2

u u x y
u, v  x y x y x  y2
 
2 2 2
1 1
    2  x2  y 2
x, y  v v y x x y 2

x y x y
2 2
x y
2 2

u, v 
i.e  e 2u        (2)
x, y 
x, y  u, v 
   e2u  e 2u  1
u, v  x, y 
yz zx xy u, v, w
4. If u  , v  , w  , Show that 4
x y z   x, y , z 
u u u yz z y
 2
x y z x x x
 u, v, w v v v z  zx x
Now,  
  x, y, z  x y z y y2 y
w w w y x  xy
x y z z z z2

DEPT OF MATHS, SJBIT Page 80


ENGINEERING MATHEMATICS-I 15MAT11

 u, v, w  yz   zx   xy   z  z   xy   y  x 


              
 x, y, z   x 2  y 2  z 2   x  y  z 2   z  y 
i.e
 y  z  x   y   zx 
        2 
 x  y  z   z  y 

= 4, as desired.

x, y, z 
5. If x  r sin  cos  , y  r sin  sin  , z  r cos ,show that  r 2 sin 

 r,  ,  
Now, by definition
x x x
r  
 x, y, z  y y y

 r ,  ,   r  
z z z
r  

sin  cos  r cos  cos   r sin  sin 


  x, y , z 
i.e  sin  sin  r cos  sin  r sin  cos 
r ,  ,  
cos   r sin  0

 
 sin  cos  0  r 2 sin 2  cos  
 r cos  cos 0  r sin  cos  cos  


 r sin  sin   r sin 2  sin   r cos 2  sin  
 
 r 2 sin 2  sin  cos 2   r 2 sin  cos 2  cos 2 

 
 r 2 sin  sin 2   cos 2  cos 2   r 2 sin  sin 2 

 r2 sin  cos 2
  sin 2 
 r 2 sin  , as required

DEPT OF MATHS, SJBIT Page 81


ENGINEERING MATHEMATICS-I 15MAT11

MODULE III

VECTOR CALCULUS

CONTENTS:

 Vector function……………………………………………………83

 Velocity and Accleleration………………………………………..87

 Gradient, Divergence, Curl, Laplacian Vector function……….92

 Solenoidal and Irrotational vectors……………………………..94

 Vector Identities…………...…………………….....……………..99

DEPT OF MATHS, SJBIT Page 82


ENGINEERING MATHEMATICS-I 15MAT11

Introduction:
Basically vector is a quantity having both magnitude and direction. Vector
quantities like force, velocity, acceleration etc. have lot of reference in physical and
engineering problems. We are familiar with vector algebra which gives an exposure to all
the basic concepts related to vectors.
Differentiation and Integration are well acquainted topics in calculus. In the
background of all these we discuss this chapter vector calculus comprising vector
Differentiation. Many concepts are highly significant in various branches of engineering.

Basic Concepts – Vector function of a single variable and the derivative


of a vector
Let the position vector of a point p(x, y, z) in space be

r  xi  yi  zk

If x, y, z are all functions of a single parameter t, then r is said to be a vector function of
 
t which is also referred to as a vector point function usually denoted as r = r (t). As the
parameter t varies, the point P traces in space. Therefore

r = x(t) i+y (t) j+z(t) k
is called as the vector equation of the curve.

dr  dx dy dz
 r ' (t )  i  j k
dt dt dt dt
Is a vector along the tangent to the curve at P.
If t is the time variable,


 dr
 gives the velocity of the particle at time t.
dt


  
d d  dr  d 2r


Further a    represents the rate of change of velocity
dt dt  dt  dt 2
 
and is called the acceleration of the particle at time t.

1.
d
dt   

  
c1 r1 (t )  c2 r2 (t )  c1 r1'(t )  c2 r2 (t ) where c1 , c2 are constants.
 

2. d  F . G   F . d G  d F . G
   

dt   dt dt

DEPT OF MATHS, SJBIT Page 83


ENGINEERING MATHEMATICS-I 15MAT11
 
d    dG d F 
F G   F .  G
3. dt   dt dt
   
Where F  F (t ) and G  G (t ).
Gradient, Divergence, Curl and Laplacian:
 
If  is scalar function A is vector function A  a1i  a2 j  a3k then
  
1. ie., grad     i j k
x y z

2. If A  a1i  a2 j  a3k ,

      
div A  . A   i  j  k  .  a1i  a2 j  a3k 
 x y z 
  a a a
div A  . A  1  2  3
x y z


3. If A  a1i  a2 j  a3k ,
i j k
 
     
div A    A   .
 xy z 
 
 a1 a2 a3 
a  a2    a3  a1  a a 
i 3   j   k 2  1 
 y z   x z   x y 

 2  2  2
4. Laplacian of    2   
x 2 y 2 z 2

  
 2 A 2 A 2 A

5. Laplacian of A   A  2  2  2
2

x y z

Important points:
 

ˆ ˆ ˆ dr d2 r
1. If r  x(t )i  y (t ) j  z (t )k , then is velocity and is acceleration.
dt dt 2

DEPT OF MATHS, SJBIT Page 84


ENGINEERING MATHEMATICS-I 15MAT11

  
d r  dT
 dt 
2. The unit tangent vector Tˆ    and unit normal vector is nˆ  ds where
 Tˆ
d r
dt

ds
Tˆ  .
dt
 
3. If A and B are any two vector and  is angle between two vectors, then
   
 A.B 
  cos 1     .
 A. B 
 
 
 
4. Component of a vector (velocity or acceleration) F along a given vector C is the

 
c
resolved part of F given by F . nˆ where nˆ  
.
c



5. Component of a vector F along normal to the c is given by


  

F  resolved part of acceleration along c  F   F . c  .cˆ .
 


1. Find the unit tangent vector to the curve r  cos tiˆ  sin tjˆ  tkˆ.

Soln: Given the space curve r  cos tiˆ  sin tjˆ  tkˆ

 dr
T    sin tiˆ  cos tjˆ  kˆ
dt

T  cos 2 t  sin 2 t  1  1  1  2

Therefore, the unit tangent vector to the given curve at any point is


 sin tiˆ  cos tjˆ  kˆ
T
Tˆ   
2

1
2

 sin tiˆ  cos tjˆ  kˆ 
T

2. Find the unit normal vector to the curve r  4sin tiˆ  4cos tjˆ  3tkˆ.

Soln: Given r  4sin tiˆ  4cos tjˆ  3tkˆ

DEPT OF MATHS, SJBIT Page 85


ENGINEERING MATHEMATICS-I 15MAT11

dr 
T   4 cos tiˆ  4sin tjˆ  3kˆ
dt


T  16 cos 2 t  sin 2 t  9  25  5 
Therefore, the unit tangent vector to the given curve at any point t is


4 cos tiˆ  4sin tjˆ  3kˆ 
ˆ T
T  
5
1
 4 cos tiˆ  4sin tjˆ  3kˆ
5
 
T

 
 dTˆ   dT 
ˆ
dT 
ˆ   
dt   1  1 4sin tiˆ  4 cos tjˆ 
ds 
 dt
ds

 
 
5  5
 


   dr 
 dt   dt 
 
dTˆ
ds

4
25

sin tiˆ  cos tjˆ 
dTˆ
2
 4 
and
ds
   sin 2 t  cos 2 t 
 25 
4

25

The unit normal vector to the given curve is


 dTˆ 

nˆ 



ds  (4 / 25) sin tiˆ  cos tjˆ

  sin tiˆ  cos tjˆ

dTˆ (4 / 25)
ds

Find the angle between tangents to the curve x  t , y  t , z  t at t=2 and


2 3 4
3.
t=3.

Soln: Define the position vector r  x(t )iˆ  y (t ) ˆj  z (t )kˆ

r  r 2iˆ  t 3 ˆj  t 4 kˆ

 dr
i.e., T  2tiˆ  3t 2 ˆj  4t 3 kˆ
dt
 
AT t 2  4iˆ  12 ˆj  32kˆ  4(iˆ  3 ˆj  8kˆ)

DEPT OF MATHS, SJBIT Page 86


ENGINEERING MATHEMATICS-I 15MAT11


 A  16(1  9  64)  4 74
 
B T t 3  6iˆ  27 ˆj  108kˆ  3(2iˆ  9 ˆj  36kˆ)

A  9(4  81  1296)  3 1381
 
Let  be the angle between two vectors A and B , then

 

  cos 1 
 
A. B 
  cos 1 
  
 4 iˆ  3 ˆj  8kˆ .3 2iˆ  9 ˆj  36kˆ   .

   4 74  3 1381 
 A . B   
 
 2  27  248 
 cos 1    cos  0.8665 
1

 74  1381 
  30 

A particle moves along the curve x  1  t , y  1  t , z  2t  5. Determine its


3 2
4.
velocity and acceleration. Find the component of velocity and acceleration at t=1 in
the direction 2iˆ  ˆj  2kˆ
Soln: Given the position vector

r  (1  t 3 )iˆ  (1  t 2 ) ˆj  (2t  5)kˆ


dr
the velocity , v   (3t 2 )iˆ  (2t ) ˆj  (2)kˆ
dt

d2 r 
and acceleration a   (6t 3 )iˆ  2 ˆj
dt 2

at t  1, v  3iˆ  2 ˆj  2kˆ

a  6iˆ  2 ˆj
Therefore, the component of velocity vector in the given direction

a  2iˆ  ˆj  2kˆ.
 (2iˆ  ˆj  2kˆ) 6  2  4
v . nˆ  (3iˆ  2 ˆj  2kˆ).  0
4 1 4 3
The normal component of acceleration in the given direction

 (2iˆ  ˆj  2kˆ) 12  2 10


a . nˆ  (6iˆ  2 ˆj ).   .
4 1 4 3 3

DEPT OF MATHS, SJBIT Page 87


ENGINEERING MATHEMATICS-I 15MAT11

5. A particle moves on the curve x  2t 2 , y  t 2  4t , z  3t  5, where t is the time.


Find the components of velocity and acceleration at time t=1 in the direction
iˆ  3 ˆj  2kˆ.

Soln: The position vector at any point (x,y,z) is given r  xi  yj  zk , but

r  2t 2iˆ  (t 2  4t ) ˆj  (3t  5)kˆ
Therefore, the velocity and acceleration are

 dr
v  4tiˆ  (2t  4) ˆj  3kˆ
dt

 d2 r
a  4iˆ  2 ˆj
dt 2

at t  1, v  4iˆ  2 ˆj  3kˆ

a  4iˆ  2 ˆj
Therefore the component of velocity in the given direction iˆ  3 ˆj  2kˆ is
 (iˆ  3 ˆj  2kˆ) 16
v . nˆ  (4iˆ  2 ˆj  3kˆ). 
1 9  4 14
Since dot product of two vector is a scalar.
The components of acceleration in the given direction iˆ  3 ˆj  2kˆ is
 (iˆ  3 ˆj  2kˆ) 2
a . nˆ  (4iˆ  2 ˆj ). 
1 9  4 14

Unit Normal Vector:



n 
nˆ    where nˆ    Normal vector
n 

Directional Derivative (D.D.):



If a is any vector and  is any scalar point function then Directional Derivative

  .a
(D.D.)=  .a  
a
Maximum directional derivative (Normal derivative)

The directional derivative will be the maximum in the direction  (i.e., a   )

2

and the maximum value of directional derivative =  .  = =  .
 

DEPT OF MATHS, SJBIT Page 88


ENGINEERING MATHEMATICS-I 15MAT11

Note : The maximum directional derivative is also called normal derivative i.e., Normal
derivative = 

Equations of Tangent plane and Normal Line:

Let  ( x, y, z )  c be any given surface and ( x1 , y1 , z1 ) be a point on it, then


(i) Equation of tangent plane to  ( x, y, z )  c at P( x1 , y1 , z1 ) is
        
(x  x )    (y  y )   ( z  z1)   0
1  x  1  y p  z p
p
(ii) Equation of normal line to  ( x, y, z)  c at P( x1 , y1 , z1 ) is
 x  x1    y  y1    z  z1 
        
   y   
 x p  p  z p

Ex. 1: If   x3 y3z3 , find  at (1,2,1) along ˆi+2j+2k


ˆ ˆ

Soln: Given   x3 y3 z3 , then


 ˆ  ˆ  ˆ
 = i+ j+ k
x y z
ˆ x3 y 2 z 3 )jˆ  (3x3 y3 z 2 )kˆ
=(3x 2 y3 z 3 )i+(3
 (1,2,1)  24iˆ  12ˆj  24kˆ
 
Let a  ˆi  2ˆj  2kˆ  a

 1 4  4  3

a ˆi  2ˆj  2kˆ
aˆ  

3
a

 at (1,2,1) along the vectors a is

ˆi  2ˆj  2kˆ 
 

ˆ ˆ ˆ
 . a = 24i  12 j  24k .
3
1 96
 (24  24  48) 
3 3

 . a = 32

DEPT OF MATHS, SJBIT Page 89


ENGINEERING MATHEMATICS-I 15MAT11

Ex. 2: Find the unit normal vector to the surface xy z  4 at the point
3 2

(-1,-1,2).
  xy 3 z 2
  
Soln: Let  = ˆi+ ˆj+ kˆ  y3 z 2 )i+3 ˆ x 2 z 2ˆj  2 xy3 zkˆ
x y z
and   4iˆ  12ˆj  4kˆ
( 1, 1,2)

Therefore, the normal vector to the given surface is



n   4iˆ  12ˆj  4kˆ
( 1, 1,2)

n  16  144  16  176  4 11

 
 n 1
n  
 ˆi  3jˆ  kˆ
n 11

Ex. 3: Find the angle between the normals to the surface xy  z at the points (1, 4, 2)
2

and (-3,-3, 3).


Soln: Let   x, y, z   xy  z 2 be the surface
The normal to the surface is
  
 =iˆ (xy-z 2 )+jˆ (xy-z 2 )+kˆ (xy-z 2 )
x y z
 =yiˆ  xˆj  2 zkˆ
 ˆ ˆ ˆ
 4i+j-4k
(1,4,2)

 ˆ ˆ ˆ
 3i-3j+6k
( 3, 3,3)

Are normals to the surface at (1, 4, 2) and (-3,-3,3).


Let  be the angle between the normals

cos =
 
n1 . n2

 4iˆ  ˆj-4kˆ  .  -3i-3j ˆ
ˆ ˆ  6k
 
n1 n2 16  1  16 9  9  36

12  3  24

33 54
39

33 54
 39 
  cos 1   
 33 54 

DEPT OF MATHS, SJBIT Page 90


ENGINEERING MATHEMATICS-I 15MAT11

Ex.4: Find the directional derivative of   x 2 yz  4 xz 2 at the point


(-1,-2,-1) in the direction of the vector 2iˆ - ˆj- 2k.
ˆ
 =x 2 yz + 4xz 2
 2 ˆ  (x 2 yz+4xz 2 )ˆj+  (x 2 yz+4xz 2 )kˆ
 = (x yz+4xz 2 )i+
Soln: Given x y z
 (2 xyz  4 z 2 )iˆ  ( x 2 z )ˆj+(x 2 y+8xz )kˆ
 (1,2,1)  8iˆ - ˆj-10kˆ
The directional derivative of  at the point (1,-2,-1) in the direction of vector
2iˆ - ˆj- 2k.
ˆ is

ˆ  2iˆ - ˆj- 2k
ˆ  1 37
 (8iˆ - ˆj-10k).    16  1  20  
 4 1 4  3 3
 
Ex.5: Find the directional derivative of   xy 2  yz 3 at the point (1,-2,-1) in the
direction of the normal to the surface x log z  y 2  4at (1, 2,1).
  xy 2  yz3

Soln: Given
   
  y 2iˆ  2 xy  z 3 ˆj  3 yz 2 kˆ

 (1, 2, 1)  4iˆ  5 ˆj  5kˆ

Let   x log z  y 2
1
Therefore the normal vector to the surface x log z  y 2  4 is 1
x
 =logziˆ -+(-2y)jˆ    kˆ
1 z
  1
and n    log(1)iˆ  (2  2)ˆj     kˆ   4ˆj  kˆ
1 ( 1, 2,1)  1

n  16  1  17


nˆ 
n


1
17

4ˆj  kˆ 
n

Therefore, the directional derivative of the surface   xy 2  yz 3 at (1,-2,-1)


in the direction normal to the surface
x log z  y  4at (1, 2,1). is
2


.nˆ  4iˆ  5 ˆj  5kˆ .1
17

4 ˆj  kˆ 
1

17
 20  5 
25
17
.

DEPT OF MATHS, SJBIT Page 91


ENGINEERING MATHEMATICS-I 15MAT11

Ex.6: Find the equation of tangent plane and normal to line to the surface x log
z  y 2  4at the point (1, 2,1).
Soln: Let   x log z  y 2 .
 
=logz  =0
x x ( 1,2,1)

 
=  2y  =4
Therefore y y ( 1,2,1)

 x 
=  = 1
z z z ( 1,2,1)

Thertefore, equation of the tangent plane is


        
(x  x )  +( y  y )  +( z  z )   0
1  x  1  y  1  z p
p p
Hence ( x , y , z )  ( 1, 2,1), then
1 1 1
( x  1)0  ( y  2)( 4)  ( z  1)( 1)  0
 4 y  8  z 1  0
4y  z  9  0
And the equation of the normal line is
 x  x1    y  y1    z  z1    x  1   y  2   z  1
         0 4 1
     
 x   y   z 
Properties of divergence:
   
1.Prove that div  A B   div A div B
 
   
Or .  A B   . A . B
 
 
1. Pr oof: Let A  A1iˆ  A2 ˆj  A3kˆ, B  B1iˆ  B2 ˆj  B3kˆ
 
 A B    A1  B1  iˆ   A2  B2  ˆj   A3  B3  kˆ
 
  
   
.  A B    A1  B1    A2  B2    A3  B3 
  x y z
 A A A   B B B 
  1  2  3  1  2  3 
 x y z   x y z 
   
.  A B   . A . B
 

DEPT OF MATHS, SJBIT Page 92


ENGINEERING MATHEMATICS-I 15MAT11

2. Prove that
  
 

div   A    grad  . A   div A 
   
  
 
Or .   A      . A   . A 
   

Pr oof: Let A  A1iˆ  A2 ˆj  A3kˆ

then ,  A   A1iˆ   A2 ˆj   A3kˆ
    
.   A     A1     A2     A3  by the property,
  x y z
A  A  A 
  1  A1   2  A2   3  A3
x x y y z z
 A A A    
   1  2  3   A1  A2  A3
 x y z  x y z
   ˆ  ˆ 
 
 
 
   . A   A1iˆ  A2 ˆj  A3kˆ .  iˆ 
 x y
j k
y 
  
 
.   A      . A     . A 
   

Properties of curl :
1. Prove that curl:
  
  
 A1  B1   curl A curl B
 
  
  
Or    A1  B1     A   B
 
 
Pr oof: Let A  A1iˆ  A2 ˆj  A3 kˆ, B  B1iˆ  B2 ˆj  B3 kˆ
 
Therefore, A B   A1  B1  iˆ   A2  B2  ˆj   A3  B3  kˆ

iˆ ˆj kˆ
    
   A B  
  x y z
A1  B1 A2  B2 A3  B3

iˆ ˆj kˆ iˆ ˆj kˆ
     
 
x y z x y z
A1 A2 A3 B1 B2 B3
  
  
   A1  B1     A   B
 

DEPT OF MATHS, SJBIT Page 93


ENGINEERING MATHEMATICS-I 15MAT11

2. If A is a vector function and  is a scalar function then

   
 
curl   A     curl A   grad  A
   
   
 
Or     A       A     A
   

Pr oof: Let A  A1iˆ  A2 ˆj  A3 kˆ

then ,  A   A1iˆ   A2 ˆj   A3kˆ
iˆ ˆj kˆ
      
  A     A3    A3  iˆ
x y z  y z 
 A1  A2  A3
 A3  A2  ˆ
    A3   A2
z 
i
 y y z
  A A2     ˆ
    3    A3  A2 i
  y z  y z 
 A A2 ˆ    ˆ
   3  i    A3  A2 i
 y z   y z 
iˆ ˆj kˆ iˆ ˆj kˆ
     
 
x y z x y z
A1 A2 A3 A1 A2 A3

 
 
   A      A     A
 
Laplacian: The Laplacian operator  2 is defined by
2 2 2
     2  2  2
2

x y z

Irrotational Vector (or Conservative Force Field): A vector field F is said to be

irrotational vector or conservative force field or curl free vector if  F  0 or curl

F 0.

Scalar Potential: A vector field F which can be derived from the scalar field  such

that F   is called conservative force field and  is called scalar potential.

DEPT OF MATHS, SJBIT Page 94


ENGINEERING MATHEMATICS-I 15MAT11

Solenoidal Vector Function: A vector A is said to be solenoidal vector or divergence
 
free vector if div A  . A  0.

Curl of a vector function: If A is any vector function differentiable at each point
  
(x,y,z) then curl of A is denoted by curl A or  A and it is defined by
iˆ ˆj kˆ
    
curl A    A 
x y z
A A A
1 2 3
   A A   A A   A A 
curl A    A   3  2  iˆ   3  1  ˆj   2  1  kˆ
 y z   x z   x y 
 
Hence, curl of a vector function is a vector.

 
 
Ex.1: If F  grad x3 y  y 3 z  z 3 x  x 2 y 2 z 2 then find div F at (1, 2,3).
Soln: Given


F  grad x3 y  y3 z  z 3 x  x 2 y 2 z 2 

  x3 y  y 3 z  z 3 x  x 2 y 2 z 2 

     
F  3x3 y  z 3  2 xy 2 z 2 iˆ  x3  3 y 2 z  2 x 2 yz 2 ˆj  y 3  3z 2 x  2 x 2 y 2 z kˆ


div F  6 xy  2 y 2 z 2  6 yz  2 x 2 z 2  6 zx  2 x 2 y 2

 F  12  72  36  18  18  8   32.
1, 2,3
  
Ex.2 : If F =(x+y+1) iˆ  ˆj  ( x  y)kˆ, then show that F .curl F =0.

Soln: Given F =(x+y+1) iˆ  ˆj  ( x  y)kˆ,
iˆ ˆj kˆ
    
curl F    F 
x y z
x  y  1 1  ( x  y)

DEPT OF MATHS, SJBIT Page 95


ENGINEERING MATHEMATICS-I 15MAT11

 (1  0)iˆ  (1  0) ˆj  (0  1) kˆ



curl F  iˆ  ˆj  kˆ
 
 F .curl F   x  y  1 iˆ  ˆj  ( x  y)kˆ  . iˆ  ˆj  kˆ 
   x  y  1  1  ( x  y )
 0

Ex.3: If F  (ax  3 y  4 z )iˆ  ( x  2 y  3z ) ˆj  (3x  2 y  z )kˆ is solenoidal, find ;‟a‟.
Soln: Given

F  (ax  3 y  4 z )iˆ  ( x  2 y  3z ) ˆj  (3x  2 y  z )kˆ, then
   
. F  (ax  3 y  4 z )  ( x  2 y  3z )  (3x  2 y  z )  a  2  1
x y z

. F  a  3

Since the vector field is solenoidal therefore . F  0, then
a 3  0  a  3

Ex.4: Find the constants a,b,c such that the vector



F  ( x  y  az )iˆ  ( x  cy  2 z )kˆ  (bx  2 y  z ) ˆj is irrotational.

Soln: Given F  ( x  y  az )iˆ  ( x  cy  2 z )kˆ  (bx  2 y  z ) ˆj

Since the vector field is irrotational, therefore  F  0 .

iˆ ˆj kˆ
   
curl F 
x y z
x  y  az bx  2 y  z x  cy  2 z
 (c  1)iˆ  (1  a ) ˆj  (b  1)kˆ
i.e., (c  1)iˆ  (1  a ) ˆj  (b  1)kˆ  0
This is possible only when,
c-1=0, 1-a=0, b-1=0  a=1, b=1, c=1.

DEPT OF MATHS, SJBIT Page 96


ENGINEERING MATHEMATICS-I 15MAT11


n2
Ex.5: Prove that 1. r  nr
n
r,
Soln: We have by the relation x=rcos  , y=r sin  .
By definition
  
r n 
x
 
r n iˆ 
y
r n ˆj  
z
rn   kˆ
r ˆ r ˆ r ˆ
 nr n 1 i  nr n 1 j  nr n 1 k
x y z
But r 2  x 2  y 2
r r x r y r z
 2r  2x   lll ly  , 
x x r y r z r
x  y z
 nr n 1   iˆ  nr n 1   ˆj  nr n 1   kˆ
r r r
 nr n  2 xiˆ  nr n  2 yjˆ  nr n  2 xzkˆ


 nr n  2 xiˆ  yˆj  zkˆ 

 nr n  2 r

2
Ex.6: Prove that 2 f (r )  f (r )  f (r ), wherer r 2  x 2  y 2  z 2.
r
Soln: Given r 2  x2  y 2  z 2 .
r r x r y r z
 2r  2x   similarly  , 
x x r y r z r
 2 2  r
Let   f ( x)   2     f ( x)    f (r ) 
x 2 x 2 x  x 

 2  2  2  2
Where    
x 2 x 2 y 2 z 2
  x   xf (r ) 
   f (r )   
x  r x  r 
1   r 
  r x  xf (r )  xf (r ) x 
r2
1   r  x
  r  xf (r ) x  1. f (r )   xf (r ) r 
r2    
1   x  x
2 
  r  xf (r )  f (r )   f (r ) 
r 2   r  r 

DEPT OF MATHS, SJBIT Page 97


ENGINEERING MATHEMATICS-I 15MAT11

1  x2 
  x 2 f (r )  rf (r )  
f (r ) 
r2  r 

1  2  x2  
  x f (r )   r   f (r ) 
r 2   r  
  
1   x2   1   y2  
  x 2 f (r )   r   f (r )    x 2 f (r )   r   f (r ) 
r2   r   r2   r  
       
1  2  z2  
 x f ( r )   r   f ( r ) 
r 2   r  
  
 2 2 2
  x 2  y 2  z 2  f (r )  1  r  x  r  y  r  z  f (r )
1
 
r2 r 2  r r r 


1 2
r2
r f (r ) 
1 

r2 
3r 
1 2
r  
x  y 2  z 2  f (r )



1  2
r2 

r f (r )  

1 
 r2   3r 
1 2 
r 
 
r  f (r )

1  2
r f (r )  
1

  3r  r  f (r )
r2   r2
3
 2 f ( r )  f ( r )  f ( r )
r


  
Ex.7: Find the constants „a‟ and „b‟ so that F  axy  z3 iˆ  3x 2  z ˆj  bxz 2  y kˆ   

irrotational and find  such that F   .

Soln: Given F  (axy  z3 )iˆ  (3x 2  z ) ˆj  (bxz 2  y )kˆ
 
Since F is irrotational i.e.,  F  0 .

iˆ ˆj kˆ
  
0
x y z
axy  z 3 3x 2  z bxz 2  y

i.e., (1  1)iˆ  (bz 2  3 z 2 ) ˆj  (6 x  ax) kˆ  0


i.e.,  z 2 (b  3) ˆj  (6  a)  0
which holds good if any only if b-3=0 and 6-a=0  a =6 and b=3.
DEPT OF MATHS, SJBIT Page 98
ENGINEERING MATHEMATICS-I 15MAT11

Also given that   F
  ˆ  ˆ  ˆ 
 i
  x y z   
j  k   6 xy  z 3 iˆ  (3x 2  z ) ˆj  (3xz 2  y )kˆ


  6 xy  z 3    3x 2 y  xz 3  f ( y, z )
x 1

 3x 2  z    3x 2 y  yz  f ( x, z )
y 2

 3xz 2  y    xz 3  yz  f ( x, y )
z 2
Hence   3x 2 y  xz 3  yz.

DEPT OF MATHS, SJBIT Page 99


ENGINEERING MATHEMATICS-I 15MAT11

DEPT OF MATHS, SJBIT Page 100


ENGINEERING MATHEMATICS-I 15MAT11

DEPT OF MATHS, SJBIT Page 101


ENGINEERING MATHEMATICS-I 15MAT11

DEPT OF MATHS, SJBIT Page 102


ENGINEERING MATHEMATICS-I 15MAT11

1.

2.

DEPT OF MATHS, SJBIT Page 103


ENGINEERING MATHEMATICS-I 15MAT11

DEPT OF MATHS, SJBIT Page 104


ENGINEERING MATHEMATICS-I 15MAT11

MODULE IV

INTEGRAL CALCULUS
CONTENTS:

 Introduction……………………………...………………………..106

 Reduction formulae for the integrals of sinnx, cosnx, sinmx


cosnx………………………………………………………………..107
 Evaluation of these integrals with standard limits problems…..108

DIFFERENTIAL EQUATIONS

 Solution of first order and first degree equations………………110

 Exact equations…………………………………………………...114

 Orthogonal trajectories…………………………………………121

DEPT OF MATHS, SJBIT Page 105


ENGINEERING MATHEMATICS-I 15MAT11

Reduction formula:
 /2

 sin  sin
n n
1. Reduction formula for x dx and x dx, n is a positive integer.
0

I n   sin n x dx
Let
 sin x .sin x dx   u v dx ( say )
n 1
=

We have the rule of integration by parts,


 u v dx  u  v dx   v dx.u dx
 I  sin n  1 x( cos x)     cos x   n  1 sin n  2 x.cos x dx
n
  sin n  1 x.cos x  (n  1)  sin n  2 x.cos 2 x dx
  sin n  1 x.cos x  (n  1)  sin n  2 x(1  sin 2 x) dx
  sin n  1 x.cos x  (n  1)  sin n  2 x dx  (n  1)  sin n x dx
i.e., I  sin n  1 x.cos x  (n  1) I  (n  1) I
n n2 n
i.e., I  1  (n  1)    sin n  1 x.cos x  (n  1) I
n n2
n  sin n  1 x.cos x n  1
 I   sin x dx   I
n n n n2

This is the required reduction formula.

 /2

 co s x  cos
n n
2. . Reduction formula for and x dx,
0
Where n is a positive integer.
I n   cos n x dx
Let
=  cos n 1 x .cos x dx

DEPT OF MATHS, SJBIT Page 106


ENGINEERING MATHEMATICS-I 15MAT11

 I  cos n  1 x.sin x   sin x  n  1 cos n  2 x( sin x) dx


n
  cos n  1 x.cos x  (n  1)  cos n  2 x.sin 2 x dx
 cos n  1 x.sin x  (n  1)  cos n  2 x(1  cos 2 x) dx
 cos n  1 x.sin x  (n  1)  cos n  2 x dx  (n  1)  cos n x dx
I  cos n  1 x.sin x  (n  1) I  (n  1) I
n n2 n
i.e., I  1  (n  1)   cos n  1 x.sin x  (n  1) I
n n2
n cos n  1 x.sin x n  1
 I   cos x dx   I
n n n n2
 /2
Next, let I   cos
n
xdx
n
0
 /2
 cos n  1 x.sin x  n 1
 from (1), I     I
n  n  n n2
 0
But cos  / 2   0  sin 0.
n 1 \
Thus I  I
n n n2

 /2

 sin  sin
m n m
1. Reduction formula for x cos x dx and cos n x dx where m and n
0
are positive integers.


I m,n   sin m x cos n xdx 
=  sin
m 1
x  sin x cos x  dx   u v dx (say )
n

DEPT OF MATHS, SJBIT Page 107


ENGINEERING MATHEMATICS-I 15MAT11

we have  u v dx  u  v dx   v dx.u  dx
Here  u dx   sin x cos n dx
Put cos x  t   sin xdx  dt
t n 1 cos n 1 x
 v dx   t dt   
n
Hence
n 1 n 1
  cos x   cos n 1 x
n 1
Now I m,n 
 sin m 1 x     (m  1)sin m  2 x cos xdx
 n 1  n 1
sin m 1 x cos n 1 x m  1
i.e.,  
n 1

n 1  sin m  2 x cos n  2 xdx
m 1 n 1
sin x cos x m  1
n 1 
  sin m  2 x.cos n x.cos 2 xdx
n 1
sin m 1 x cos n 1 x m  1

n 1

n 1  sin m  2 x cos n x(1  sin 2 x)dx
m 1 n 1
sin x cos x m  1 m 1

n 1

n 1  sin m  2 x cos n xdx 
n 1  sin m x cos n xdx
m 1 n 1
 sin x cos x m  1 m 1
I m,n    I m  2, n  I m ,n
n 1 n 1 n 1
 m  1 1
i.e., I m ,n 1      sin m 1 x cos n 1 x  (m  1) I m  2, n 
 n 1  n 1
 m  1 1
I m,n      sin m 1 x cos n 1 x  (m  1) I m  2, n 
 n 1  n 1
sin m 1 x cos n 1 x m  1
 I m ,n   sin x cos xdx  
m n
 I m  2, n ......(1)
mn mn

PROBLEMS:

1. Let I   sin
4
x dx
0

f ( x)  sin 4 x and 2a   or a   / 2
f (2a  x)  sin 4 (  x)  sin 4 x  f ( x) ie., f (2a  x)  f ( x)
2a a
Thus by the property 0
f ( x)dx 2  f ( x)dx we have,
0
 /2
3 1 
I 2  sin x dx  2. . . by reduction formula.
4

0
4 2 2
Thus I=3 / 8

DEPT OF MATHS, SJBIT Page 108


ENGINEERING MATHEMATICS-I 15MAT11


2. Let I   x sin
8
x dx
0
a a
We have the property  f ( x) dx   f ( a  x) dx
0 0
 
I   (  x) sin 8 (  x)dx   (  x) sin 8 xdx
0 0
 
   sin 8 xdx   x sin 8 xdx
0 0

I    sin 8 xdx  I
0
 /2
or 2 I   .2  sin
8
xdx
0

7 5 3 1 
Hence I   . . . . . , .
8 6 4 2 2
Thus by reduction formula
35 2
I=
256


3. Let I   x sin
2
x cos 4 x dx
0

I   (  x) sin (  x) cos 4 (  x) dx, by a property.
2

0

  (  x ) sin 2 x cos 4 xdx
0
 
   sin 2 x cos 4 x dx   x sin 2 x cos 4 x dx
0 0

   sin 2 x cos 4 x dx  I
0
 /2
2 I   .2 
0
sin 2 x cos 4 x dx

(1).(3).(1) 
I  . . by reduction formula.
6 4 2 2
Thus I= 2 / 32

DEPT OF MATHS, SJBIT Page 109


ENGINEERING MATHEMATICS-I 15MAT11

2.

Introduction:
Many problems in all branches of science and engineering when analysed for
putting in a mathematical form assumes the form of a differential equation.
An engineer or an applied mathematician will be mostly interested in obtaining a solution
for the associated equation without bothering much on the rigorous aspects. Accordingly
the study of differential equations at various levels is focused on the methods of solving
the equations.
Preliminaries:
Ordinary Differential Equation (O.D.E)
If y = f (x) is an unknown function, an equation which involves atleast one derivative of
y, w.r.t. x is called an ordinary differential equation which in future will be simply
referred to as Differential Equation (D.E).
The order of D.E is the order of the highest derivative present in the equation and the
degree of the D.E. is the degree of the highest order derivative after clearing the
fractional powers.
Finding y as a function of x explicitly [y = f (x)] or a relationship in x and y satisfying the
D.E. [f (x, y)= c] constitutes the solution of the D.E.
Observe the following equations along with their order and degree.

DEPT OF MATHS, SJBIT Page 110


ENGINEERING MATHEMATICS-I 15MAT11
dy
1  2x [order = 1, degree = 1]
dx
2
 dy  dy
2   3 2  0 [order = 1, degree = 2]
 dx  dx

General solution and particular solution:


A solution of a D.E. is a relation between the dependent and independent
variables satisfying the given equation identically.

The general solution will involve arbitrary constants equal to the order of the D.E.

If the arbitrary constants present in the solution are evaluated by using a set of given
conditions then the solution so obtained is called a particular solution. In many physical
problems these conditions can be formulated from the problem itself.

Note : Basic integration and integration methods are essential prerequisites for this
chapter.

Solution of differential equations of first order and first degree


Recollecting the definition of the order and the degree of a D.E., a first order and
first degree equation will be the form
dy
 f ( x, y ) or M(x,y)dx+N(x,y)dy=0
dx
We discuss mainly classified four types of differential equations of first order and first
degree. They as are as follows:
 Variables separable equations
 Homogenous equations
 Exact equations
 Linear equations
Variables separable Equations:
If the given D.E. can be put in the form such that the coefficient of dx is a
function of the variable x only and the coefficient of dy is a function of y only then the
given equation is said to be in the separable form.
The modified form of such an equation will be,
P (x) dx + Q (y) dy = 0

DEPT OF MATHS, SJBIT Page 111


ENGINEERING MATHEMATICS-I 15MAT11

This is the general solution of the equation.


dy y  x2
Example 1: Solve  xe given that y(0)=0
dx
dy dy
 xe y  x or  xe y e x
2 2

Soln: dx dx

put - x 2  t 2 xdx  dt or  xdx  dt


 y  et dt  c
Hence we have,  e 
dy  x2
i.e.  xe dx by separating theVariables
ey
  e y dy   xe  x dx  0
2

i.e  e  y   xe  x dx  c
2

The general solution becomes

et
i.e.  e y  c
2
e x
2

or  e y  c is the general solution.


2
Now we consider y (0) =0 That is y =0 when x =0,

1 1
 1  c or c = 
2 2
Now the general solution becomes
e x
2
1
 e y  
2 2
This is the required solution.

DEPT OF MATHS, SJBIT Page 112


ENGINEERING MATHEMATICS-I 15MAT11

dy
Example –2 solve : xy  1  x  y  xy
dx
dy
 xy  1  x  y  xy
dx
dy
i.e., xy  (1  x)  y (1  x )
dx
dy
i.e., xy  (1  x)(1  y )
dx
ydy 1  x
or  dx by separating the variables.
1 y x
y 1 x
  dy   dx  c
1 y x
(1  y )  1 1
or  dy   dx   1dx  c
1 y x
1
ie.,  1dy   dy  log x  x  c
1 y
i.e y  log y  log x  x  c
or  y  x   log yx  c is the required solution

dy dy
Solve : y  x  y2 
Example – 4 : dx dx
>> Rearranging the given equation we have,
dy
y  y 2   x  1
dx
dx dy
or  
 x  1 
y  y2 
dy
i.e log  x  1            (1)
y 1  y 

We have to employ the method of partial fractions for the second term of the above.
1 A B
Let  
y 1  y  y y  1
 1  A  y  1  By

Put y=0,1 A=-1 and B=1

DEPT OF MATHS, SJBIT Page 113


ENGINEERING MATHEMATICS-I 15MAT11

dy dy dy
    
y 1  y  y 1  y 
dy 1  y 
 y 1  y    log y  log 1  y   log y

Using this result in (1) we get,


 1 y 
log  x  1  log  c
 y 
  x  11  y  
or log    log k
 y 
  x  11  y   ky is the required solution.
Example – 5 :
Solve:
dy
tan y  cos( x  y)  cos( x  y )
dx
 The given equation on expanding terms in the R.H.S. becomes
dy
tan y  cos x cos y  sin x sin y  cos x cos y  sin x sin y
dx
dy
ie., tan y  2 cos x cos y
dx
tany
or dy  2 cos x dx by separating the variables.*
cosy
  tany. sec y dy   2 cos x dx  c
sec y  2sin x  c is the required solution.

Exact Differential Equations:


The differential equation M(x , y) dx + N(x , y)dy=0 to be an exact equation is
M N

y x
Further the solution of the exact equation is given by
 M dx   N ( y) dy  c
Where, in the first term we integrate M(x,y) w.r.t x keeping y fixed and N(y) indicate the
terms in N with out x
(not containing x)
1.  2 2 3

3

Solve: 5x  3x y  2 xy dx  2 x y  3x y  5 y dy  0
4 2 2 4

DEPT OF MATHS, SJBIT Page 114
ENGINEERING MATHEMATICS-I 15MAT11

(Though it is evident that the equation is a homogeneous one, before solving by putting
y=vx we should check for exactness)
 Let M  5 x 4  3x 2 y 2  2 xy 3and N  2 x 3 y  3x 2 y 2  5 y 4
M N
 6 x 2 y  6 xy 2 and  6 x 2 y  6 xy 2
y x
M N
Since  , the given equation is exact.
y x
The solution is  M dx   N ( y ) dy  c

 5x 
 3 x 2 y 2  2 xy 3 dx   5 y 4 dy  c
4
ie.,
Thus x 5  x 3 y 2  x 2 y 3  y 5  c, is the required solution.

Solve: cos x tan y  cos( x  y) dx  sin x sec y  cos( x  y)  dy  0


2
2.
 Let M  cos x tan y  cos( x  y ) : N  sin x sec 2 y  cos( x  y )
M N
  cos x sec 2 y  sin( x  y );  cos x sec 2 y  sin( x  y )
y x
M N
Since  , the given equation is exact.
y x
The solution is  M dx   N ( y ) dy  c
ie.,  cos x tan y  cos( x  y)dx   0 dy  c
Thus sin x tan y  sin( x  y )  c, is the required solution.

dy y cos x  sin y  y
3. Solve:  0
dx sin x  x cos y  x
>> The given equation is put in the form,
 y cos x  sin y  y  dx   sin x  x cos y  x  dy  0.
Let M  y cos x  sin y  y and N  sin x  x cos y  x
M N
  cos x  cos y  1 and  cos x  cos y  1
y x
M N
Since  , the given equation is exact.
y x
The solution is  M dx   N ( y ) dy  c
ie.,   y cos x  sin y  y  dx   0 dy  c
Thus y sin x  x sin y  xy  c, is the required solution.

DEPT OF MATHS, SJBIT Page 115


ENGINEERING MATHEMATICS-I 15MAT11

4. 
Solve: ye xy dx  xe xy  2 y dy  0
 Let M  ye xy , N  xe xy  2 y
M N
 ye xy x  e xy ;  xe xy y  e xy
y x
M N
Since  , the equation is exact.
y x
solution is given by  M dx   N ( y ) dy  c
ie.,  ye xy dx   2 y dy  c
e xy
ie., y  y2  c
y
Thus e xy +y 2  c, is the required solution.
5. Solve:  y(1  1/ x)  cos y  dx   x  log x  x sin y  dy  0
 Let M  y (1  1/ x)  cos y and N  x  log x  x sin y
M N
  1  1/ x  sin y and  1  1/ x  sin y
y x
M N
Since  , the given equation is exact.
y x
The solution is  M dx   N ( y ) dy  c
ie.,   y(1  1/ x)  cos y dx   0 dy  c
Thus y ( x  log x)  x cos y  c, is the required solution.

Equations reducible to the exact form:

Integrating factor: Type-1:

Suppose that, for the equation M dx + N dy =0


M N
 , then we take their difference.
y x
M N
The difference  should be close to the expression of M or N.
y x
1  M N  1  M N 
If it is so, then we compute    or   
M  y x  N  y x 
1  M N  1  M N 
If     f ( x) or     g ( y)
N  y x  M  y x 

Then e or e 
f ( x ) dx  g ( y ) dy
is an integrating factor.

DEPT OF MATHS, SJBIT Page 116


ENGINEERING MATHEMATICS-I 15MAT11

The following basic results will be useful :


log x n log x
(i) e  x (ii) e  xn

1.  
Solve: 4 xy  3 y  x dx  x  x  2 y  dy  0
2

 Let M  4 xy  3 y 2  x and N  x  x  2 y   x 2  2 xy
M N
 4 x  6 y and  2 x  2 y.  The equation is not exact 
y x
M N
Consider   2 x  4 y  2( x  2 y )....close to N .
y x
1  M N  2( x  2 y ) 2
Now      f ( x)
N  y x  x( x  2 y ) x

Hence e 
f ( x)dx
is an integrating factor.
2
f ( x)dx 
f dx
2log x log( x 2 )
ie., e  e x e e  x2
Multiplying the given equation by x 2 we now have,
M  4 x3 y  3x 2 y 2  x 3 and N  x 4  2 x 3 y
M N
 4 x3  6 x 2 y and  4 x3  6 x 2 y
y x
Solution of the exact equation is  M dx   N ( y) dy  c

  4 x y  3x 
y 2  x 3 dx   0 dy  c
3 2
ie.,
x4
Thus x 4 y  x 3 y 2   c, is the required solution.
4

2. Solve: y(2x-y+1)dx+x(3x-4y+3)dy=0
 Let M  y  2 x  y  1 and N  x  3x  4 y  3
ie., M  2 xy  y 2  y and N  3x 2  4 xy  3x
M N
 2 x  2 y  1,  6x  4 y  3
y x
M N
  4 x  2 y  2  2(2 x  y  1)....near to M .
y x
1  M N  2(2 x  y  1) 2
Now       g ( y)
M  y x  y (2 x  y  1) y
2
 dy
 g ( y )dy 2 log y log( y 2 )
Hence I .F  e 
y
e e e  y2

DEPT OF MATHS, SJBIT Page 117


ENGINEERING MATHEMATICS-I 15MAT11

Multiplying the given equation by y 2 we now have,


M  2 xy 3  y 4  y 3 and N  3x 2 y 2  4 xy 3  3xy 2
M N
 6 xy 2  4 y 3  3 y 2 and  6 xy 2  4 y 3  3 y 2
y x
The Solution is  M dx   N ( y ) dy  c

  2 xy 
 y 4  y 3 dx   0 dy  c
3
ie.,
Thus x 2 y 3  xy 4  xy 3  c, is the required solution.

Integrating Factor: Type-2:

If the given equation M dx +N dy=0 is of the form


yf (xy) dx+xg(xy)dy=0
1
then is an integrating factor provided Mx  Ny  0
Mx  Ny

1.  
Solve: y 1  xy  x y dx  x 1  xy  x y dy  0
2 2 2 2
 
>> The equation is of the form yf(xy)dx+xg(xy)dy=0 where,
M  yf ( xy )  y  xy 2  x 2 y3 and
N  xg ( xy )  x  x 2 y  x3 y 2

 
Now Mx  Ny  xy  x 2 y 2  x3 y3  xy  x 2 y 2  x3 y3  2 x 2 y 2 
1 1
  is the I .F .
Mx  Ny 2 x y 2
2
2 2
Multiplying the given equation with 1/ 2x y it becomes an exact equation where we now have,
1 1 y 1 1 x
M   and N   
2
2x y 2 x 2 2 xy 2 2 y 2

The solution is given by  M dx   N ( y )dy  c


 1 1 y 1
ie.,   2 x2 y 2 x  2  dx    2 y dy  c
 
 
1 1 xy 1
ie.,  log x   log y  c
2 xy 2 2 2

2. 
Solve: y xy  2 x y
2 2
 dx  x  xy  x y  dy  0
2 2

>> The equation is of the form yf(xy)dx+xg(xy)dy=0 where,


DEPT OF MATHS, SJBIT Page 118
ENGINEERING MATHEMATICS-I 15MAT11

M  xy 2  2 x 2 y3 and
N  x 2 y  x3 y 2
Now Mx  Ny  x 2 y 2  2 x3 y3  x 2 y 2  x3 y3  3x3 y3
3 3
Thus 1/ 3x y is the I.F. Multiplying the given equation by this I.F we have an exact equation
where we now have,
1 1 1 1
M  and N  
2
3x y 3x 3xy 2 3y

The solution is  M dx   N ( y ) dy  c
 1 2  1
ie.,   3x2 y 3x  dx    3 y dy  c
 
 
1 2 1
ie.,   log x  log y  c
3 xy 3 3

Integrating factor: Type-3:

If the given equation Mdx+Ndy =0 is of the form

xk1 y k2 (c1 ydx  c2 x dy)  x k3 y k4 (c3 ydx  c4 x dy)  0


Where ki and ci (i=1 to 4) are constants then x a y b is an integrating factor. The constants a and
b are determined such that the condition for an exact equation is satisfied.

1. Solve: x(4y dx+2x dy)+ y 3 (3y dx+5x dy) = 0

>> We have (4 xy  3 y 4 )dx  (2 x2  5xy3 )dy  0

Multiplying the equation by x a y b we have,


 4(b  1)  2(a  2) and 3(b  4)  5(a  1)
ie., a  2b and 5a  3b  7
By solving we get a=2and b=1

We now have, M  4 x3 y 2  3x2 y5 and N  2 x4 y  5x3 y 4

The solution is  Mdx   N ( y)dy  c

DEPT OF MATHS, SJBIT Page 119


ENGINEERING MATHEMATICS-I 15MAT11

M  4 x a  1 yb  1  3x a yb  4 and
N  2 x a  2 yb  5 x a  1 yb  3
M
 4(b  1) x a  1 yb  3(b  4) x a yb  3
y
N
 2(a  2) x a  1 yb  5(a  1) x a yb  3
x
M N
We have to find a and b such that 
y x

ie.,  3 2 2 5

 4 x y  3x y dx   0 dy  c

Thus x 4 y 2  3x 2 y5 =c, is the required solution.

2. Solve: ( y 2  2 x2 y)dx  (2 x3  xy) dy  0

>>Multiplying the given equation by x a y b we have,

M  x a yb  2  2 x a  2 yb  1 and
N  2 x a  3 yb  x a  1 yb  1
M
 (b  2) x a yb  1  2(b  1) x a  2 yb
y
N
 2(a  3) x a  2 yb  (a  1) x a yb  1
x
M N
Let us find a and b such that 
y x
 (b  2)  (a  1) and 2(b  1)  2(a  3)
ie., a  b  3 and a  b  2

By solving we get a = -5/2 and b=-1/2.


M  x 5 / 2 y3 / 2  2 x 1/ 2 y1/ 2 and
We now have,
N  2 x1/ 2 y 1/ 2  x 3 / 2 y1/ 2

The solution is  Mdx   N ( y)dy  c

DEPT OF MATHS, SJBIT Page 120


ENGINEERING MATHEMATICS-I 15MAT11

ie., 
 x 
5 / 2 y3 / 2  2 x 1/ 2 y1/ 2 dx  0 dy  c

x 3 / 2 3 / 2 x1/ 2 1/ 2
ie., y  2 y c
3 / 2 1/ 2
2 3 / 2 3 / 2
ie., x y  4 x1/ 2 y1/ 2  c
3
Thus 6 xy  y3 / x3 = k , is the required solution, where k=3c/2

Type-4 Exactness by inspection:

1. Solve : 1  y tan( xy) dx   x tan( xy) dy  0


>> The given equation can be put in the form
dx  tan ( xy )  y dx  x dy   0
ie., dx  tan( xy )d ( xy )  0
Integrating we get, x  logsec( xy)  c , being the required solution.

y dx  x dy
2. Solve:  ( x dx  y dy )  0
y2
>> The given equation is equivalent to the form,
x
d    x dx  y dy  0
 y
x x2 y 2
    c, on integration.
y 2 2

Thus   x 2  y 2   c, is the required solution.


x 1
y 2

Orthogonal trajectories

Definition: If two family of curves are such that every member of one family intersect
every member of the other family at right angles then they are said to be
orthogonal trajectories each other

DEPT OF MATHS, SJBIT Page 121


ENGINEERING MATHEMATICS-I 15MAT11

DEPT OF MATHS, SJBIT Page 122


ENGINEERING MATHEMATICS-I 15MAT11

1.

2.

DEPT OF MATHS, SJBIT Page 123


ENGINEERING MATHEMATICS-I 15MAT11

3) Find the orthogonal trajectories of the family of curves


x2 y2
  1  '  ' being the parameter  . (July 2015)
a 2 b2  
x2 y2
Soln: we have   1 ..........(1)
a 2 b2  

Differentiating the (1) equation we get,


2x 2 y dy
 2  0
a 2
b   dx
x  y dy
i.e  2  ........(2)
a 2
b   dx

x2  y2
Also from(1)  1 
a2 b2  
x2  a2  y2
  .............(3)
a2 b2  
Now , dividing (2) by(3) we get
x y dy
 2
x a
2 2
y dx
x 1 dy
 2  
x a 2
y dx
dy dx
Now let us replace by
dx dy
x 1  dx 
    
x a
2 2
y  dy 
x2  a2
or ydy   dx by separating the variables
x
dx
  ydy    xdx  a 2   c
x
y 2
x 2
i.e   a 2 log x  c is the required orthogonal trajectories
2 2

4) Find the orthogonal trajectory of the cardiods r  a 1  cos   , using the


differential equation method . (
Jan 2015 ,Jan2014)
Soln:

DEPT OF MATHS, SJBIT Page 124


ENGINEERING MATHEMATICS-I 15MAT11

dr
Differentiating the given equation w.r.t θ,we get  a sin  .Substituting for a in the given equation,we get
d
 1  cos   dr
r   .........DE of given equation
 sin   d
dr d  1  cos    2 d 
Changing to  r 2 r    r 
d dr  sin    dr 
dr
  cos ec  cot   d  0........DE of orthogonal trajectories
r
solving this equation,we get
log r  log  cos ec  cot    log sin   log c

r
 cos ec  cot    c  r 1  cos    c
sin  sin 2 
r  c 1  cos   , this is requried orthogonal trajectories.

DEPT OF MATHS, SJBIT Page 125


ENGINEERING MATHEMATICS-I 15MAT11

MODULE V

LINEAR ALGEBRA

CONTENTS:

 Elementary transformation………………………………………127

 Reduction of the given matrix to echelon and normal forms…..128

 Solution of a system of non-homogeneous equations by

Gauss Elimination method……………………………………….136

Gauss Jordan method …………………………………………141

Gauss Seidel method………………………………………. ..... 143

 Linear transformations…………………………………………..148

 Reduction to diagonal form, quadratic forms………………….157

 Reduction of quadratic form into canonical form…………..…159

 Rayleigh‟s power method to find the largest Eigen value and

the corresponding Eigen vector……………………………163

DEPT OF MATHS, SJBIT Page 126


ENGINEERING MATHEMATICS-I 15MAT11

Definition: A system of mn nos. arranged in a rectangular formation along m-rows &


n-columns & bounded by the brackets   or   is called as m by n matrix or mxn matrix
Matrix is denoted by a single capital letters A,B,C etc.
 a11 a12 .......a1n 
A   a21 a22 .......a2 n 
 am1 am 2 ......amn 
mn

Elementary operations on Matrices:


The following 3 operations are said to be elementary operations
1. Interchange of any two rows or columns.
2. Multiplication of each element of a row or column by a non-Zero scalar or constant.
3. Addition of a scalar multiple of one row or column to anpother row or column.
R3 kR1  R3
 a1 a2 a3   a1 a2 a3 
   
A  b1 b2 b3  B  b1 b2 b3 
c1 c2 c3  (ka1  c1 ) (ka2  c1 ) (ka3  c3 ) 

If a matrix A gets transferred into another matrix B by any of these transformations then
A is said to be equivalent to B written as A  B.
Echelon form or Row reduced Echelon form.
A matrix A of order mxn is said to be in a row reduced echelon form if
1. The leading element (the first non-Zero entry) of each row is unity.
2. All the entries below this leading entry is Zero.
3. The no of Zeros appearing before the leading entry in each row is greater than that
appears in its previous row.
4. The Zero rows must appear below the non-zero rows.
1 2 3 2  1 2 3 2  1 4 2 
A  0 0 1 2  0 1 2 9  0 1 0 
   
0 0 0 1 0 0 0 0  0 0 0 
Normal form of a matrix
The given matrix A is reduced to an echelon form first by applying a series of elementary
row transformations.
Later column transformations row performed to reduce the matrix to one of the
following four forms, called the normal form of A.
 Ir   Ir o 
i) Ir ii )  Ir , o  iii )   iv)   where Ir is the identity matrix of order r.
o o o 

DEPT OF MATHS, SJBIT Page 127


ENGINEERING MATHEMATICS-I 15MAT11

1 0  1 0 0 0
1 0 0  1 0 0 0    0
0 1 0  0 1 0 0 0 1   1 0 0 
    0 0  0 0 1 0
0 0 1  0 0 1 0     
0 0  0 0 0 0
 I 2   I3 0 
 I 3   I 3 , 0 0  0 0 
   

Rank of a matrix: The number of non-zero rows in echelon or normal form. It‟s in
denoted by f(A)
1. Reduce the matrix to the row reduced echelon form
1 2  2 3
0 0 1 2 
A 
 1  3 2  1
 
2 4 1 3 
Sol : R2  R2  2 R1 , R3  R3  R1 R4  R4  2 R1
1 2  2 3 1 2 2 3 
0 1 8 1   0 1 8 1 
A A
0  1 0 2  0 0 8 3 
   
0 0 3  3 0 0 3  3
R3  R3 / 8, R4  R4 R4  R4  R3
3

1 2 2 3  1 2 2 3 
0 0 
 1 8 1   1 8 1 
A 3  A  0 0 1 3 
0 0 1  8 
 8
0 0 1  3  0 0 0 11 
 8 
8
R4  R4
11
1 2 2 3 
0 1 8 1 

A
0 0 1 3 
 8
0 0 0 1 

DEPT OF MATHS, SJBIT Page 128


ENGINEERING MATHEMATICS-I 15MAT11

1  3 1 2 
2) 0 1 2 3  find rank of a matrix
3 4 1  2 
Sol : R3  R3  3R1 , R3  R3  13R2
1  3 1 2  1  3 1 2

A  0 1  
2 3  A  0 1 2 3
0 13  2  8 0 0  28  34 
 ( A)  3

8 2 1 6 
3)  2 1 0 1  find the rank
5 1 1 4 
Sol : R2  4 R2  R1 , R3  8 R3  3R1 R4  R4  R2
8 2 1 6 8 2 1 6 
0 2  1  2  0 2  1  2 
A  A
0  6 5 6 0 0 2 0 
   
0  2 3 2 0 0  2 0
R4  R4  R3
8 2 1 6 
0 2  1  2 
A
0 0 2 0
 
0 0 0 0
 ( A)  3

DEPT OF MATHS, SJBIT Page 129


ENGINEERING MATHEMATICS-I 15MAT11

1 3 2 

4) Using the elementary transformation reduce the matrix A   2  1 4  to

1  11 14 
echelon form
Sol : R2  R2  2 R1 , R3  R3  R1
1 3 2  1 3 2 

 A  2  1 4   0  7 8 
  
1  11 14  0  14 16 
R
R3  R3  2 R2 , R2  2
7
1 2 2  1 2 2 
 
 0  7 8  A 0
  1 8 
7
0 0 0   
0 0 0 

5) Applying elementary transformations reduce the following matrix to the normal form &
3 2 5 7 12 

hence find rank of matrix given 1 1 2 3 5

 
3 3 6 9 15 
Sol : R1  R2
3 2 5 7 12  1 1 2 3 5 
A  1 1 2 3 5   3 2 5 7 12 
  
3 3 6 9 15  3 3 6 9 15 
R2  R2  3R1 , R3  R3  3R1
1 1 2 3 5 
A  0  1  1  2  3 

0 0 0 0 0 
This echelon form, now we have to perform column trans to reduce to the normal form.

DEPT OF MATHS, SJBIT Page 130


ENGINEERING MATHEMATICS-I 15MAT11

c2  c2  c1 , c3  c3  2c1 c4  c4  3c1 c5  c5  5c1


1 0 0 0 0 
A  0  1  1  2  3 

0 0 0 0 0 
c3  c3  c2 , c4  c4  2c2 c3  c5  3c2
R2  R2
1 0 0 0 0  1 0 0 0 0 
A  0  1 0 0 0  A  0 1 0 0 0 
0 0 0 0 0  0 0 0 0 0 
I 0
A  2   ( A)  2
0 0 
6) By performing elementary row & column transformations, reduce the following matrix to
2  4 3 1 0 
0  2 1  4 2 
the normal form 
0 1  1 3 1
 
4  7 4  4 5

Sol : R2  R2
2  4 3 1 0  1  2 1 4 2 
0  2 1  4 2  2  4 3 1 2 
A    
0 1  1 3 1  0 1 1 3 1 
   
4  7 4  4 5 4  7 4  4 5

R2  R2  2 R1 , R4  R4  4 R2 R2  R3
1  2 1  4 2  1 2 1 4 2 
0 0 1 
9 4  2 1 1 3 1 
A    
0 1  1 3 1  0 0 1 9 4 
   
0 1 0 12  3  0 1 0 12  3 

c2  c2  2c1 , c3  c3  c1 c4  c4  4c1 , c5  c5  2c1


1 0 0 0 0 
0 1  1 3 1 
 
0 0 1 9  4 
 
0 0 0 0 0 

DEPT OF MATHS, SJBIT Page 131


ENGINEERING MATHEMATICS-I 15MAT11

c3  c3  c2 , c4  c4  3c2 , c5  c5  c2 c4  c4  9c3 c5  c5  4c3


1 0 0 0 0 1 0 0 0 0 
0 1 0 0 1  0 1 0 0 0 
   
0 0 1 9 4  0 0 1 0 0 
   
0 0 0 0 0  0 0 0 0 0 
I 0
A 3 
0 0
 ( A)  3
1 1 16 
1  1 2 5 
7) Re ducing the matrix A   int o normal form and find the rank
3 1 1 8 
 
2  2 3 7 
R2  R2  R1 , R3  R3  3R1 R4  R4  2 R1
1 1 1 6  1 1 1 6 
1  1 2 5  1  2 1  1 
A  
3 1 1 8  0  2  2  10 
   
2  2 3 7  0  4 1  5 
1
R3  R3  R2 , R4  R4  2 R2 R3  R3
3
1 1 1 6  1 1 1 6 
1  2 1  1  1  2 1  1 
A    
0 0  3  9  0 0 1 3 
   
0 0  1  3  0 0  1  3 

DEPT OF MATHS, SJBIT Page 132


ENGINEERING MATHEMATICS-I 15MAT11

R2   R2 c3  2c3  2c2 c4  2c4  c2


1 0 0 0  1 0 0 0 
0 2  1 1  0 2 1  1 
A  
0 0 1 3  0 0 1 3 
   
0 0 0 0  0 0 0 0 
c
c3  2 c  c4  3c3
2 4
1 0 0 0 
0 1 0 0 
 
0 0 1 0 
 
0 0 0 0 
I 0 
  3   ( A)  3.
0 0 

1 2  2 3 
2 5  4 6 
8) Find the rank of the matrix  
 1  3 2  2 
 
2 4  1 6 
Sol : R2  R2  2 R1 , R3  R3  R1 R4  R4  2 R1
1 2  2 3  1 2  2 3 
2 5  4 6  0 1 0 0 
A    
 1  3 2  2  0  1 0 1 
   
2 4  1 6  0 0 3 0 
1
R3  R3  R2 , R3  R4 R3  R3
3
1 2  2 3  1 2  2 3 
0 1 0 0  0 1 0 0 
A    
0 0 0 1  0 0 1 0 
   
0 0 3 0  0 0 0 1 

c2  c2  2c1 , c3  c3  2c1 c4  c4  3c1


1 0 0 0 
0 1 0 0 
  ( A)  4
0 0 1 0 
 
0 0 0 1 

DEPT OF MATHS, SJBIT Page 133


ENGINEERING MATHEMATICS-I 15MAT11

9) Find the rank of the matrix by reducing to the normal form


1 1 1 1 
1 2 3  4 
 
2 3 5  5 
 
1) 3  4  5 8 
Sol : R2  R2  R1 , R3  R3  2 R1 R4  R4  3R1

2) 1 1 1 1  1 1 1 1 
1 2 3  4  0 1 2  5 
A    
 2 3 5  5  0 1 3  7 
   
3  4  5 8   0  7  8 5 
R3  R3  R2 , R4  R4  7 R2 R4  R4  6 R3
1 1 1 1  1 1 1 1 
0 1 2  5  0 1 2  5 
A   
0 0 1  2  0 0 1 2 
   
0 0 6  30  0 0 0  18 
c2  c2  c1 , c3  c3  c1 c4  c4  c1 c3  c3  2c2 c4  c4  5c2
1 0 0 0  1 0 0 0 
0 1 2  5  0 1 0 0 
 
0 0 1  2  0 0 1 2 
   
0 0 0  18 0 0 0  18
1
c4  c4  2c3 c4  c4
18
1 0 0 0  1 0 0 0 
0 1 2 0  0 1 0 0 
   
0 0 1 0  0 0 1 0 
   
0 0 0  18 0 0 0 1 
 ( A)  4.

DEPT OF MATHS, SJBIT Page 134


ENGINEERING MATHEMATICS-I 15MAT11

1 1 1 1 
 
10) Find the value of K such that the following matrix A = 1 2 4 k  may have rank equal
1 4 10 k 2 
 
to a) 3 b) 2.

Sol : R2  R2  R1 R3  R3  R1
3)
1 1 1 1  1 1 1 1 

 
A  1 2 4 k   0 1 3  k  1 
1 4 10 k 2  0 3 9 k 2  1 
     
R3  R3  3R2
1 1 1 1 
 
0 1 3  k  1 
 
0 0 0 k  3k  2 
2

a) Rank of A can be 3 if the equivalent form of A has 3 non-Zero rows.


 
This is possible if k 2  3k  2  0
i.e, (k  1)(k  2)  0
 ( A)  3 if k  1 & k  2
4) b) Rank of A can be 2 if the equivalent form of A has 2 non-zero rows.

5)

This is possible if k 2  3k  2  0 
i.e, (k  1)(k  2)  0  k  1 or k  2
6)  ( A)  2 if k  1 & k  2
 a11a12 .........a1n : b1 
 A : B   a21a22 .........a2 n : b2 
 am1am 2 .........amn : bm 
The given sys of equation is consistent & will have unique soln.
Let us convert  A : B  into a set of equation as follows
x+ y+ z = 6
-2y + z = 7 -2y+3=-1 x+y+z=6
-3z = -9 -2y=-4 x=6-2-3=1
 z= 3 y=2 x=1

x=1, y=2, z=3is the unique soln.

DEPT OF MATHS, SJBIT Page 135


ENGINEERING MATHEMATICS-I 15MAT11

7) Solve the system of equations: x+2y=3z=0


2x+3y+z=0
4x+5y+4z=0
x+y-2z=0
1 2 3 :0 
 2 3 1 :0 
Sol :  A : B    
 4 5 4 :0 
 
1 1  2 : 0 
R2  R2  2 R1 R3  R3  4 R1 R4  R4  R1 R3  R3  3R2 R4  R4  R2
1 2 3 :0  1 2 3 :0 
0  1  5 :0  0  1  5 :0 
   
0  3  8:0  0 0 7:0 
   
0  1  5: 0  0 0 0 : 0 
 ( A)  3
  A : B  3  n  3
Hence the system is consistent & will have trivial soln x=0 y=0 z=0

8) Does the following system of homogenous equations possess a non-trivial


solutions? If so find them
x1  x2  x3  x4  0
x1  x2  2 x3  x4  0
3 x1  x2  x4  0
1 1  1 1 :0 
 A : B   1  1 2  1 :0 
3 1 0 1 :0 
R2  R2  R1 R3  R3  3R1 R3  R2
1 1  1 1 :0  1 1  1 1 :0 
 0  2 3  2 :0   0  2 3  2 :0 
0  2 3  2 :0  0 0 0 0 :0 

Gauss elimination method:


The simplest method of solving systems of the form (1) of section 5.2 is the
elimination method.
The Working Rule for the method is as given below.

DEPT OF MATHS, SJBIT Page 136


ENGINEERING MATHEMATICS-I 15MAT11

Working rule:

Step1: Reduce the augmented matrix (A:B) to the form where A is in echelon form or in
upper triangular form, by employing appropriate elementary row operations.

Step2: Write the linear equations associated with the reduce form obtained in Step 1. Let
the number of equations in this reduced system be equal to r, If r=n, then the reduced
system yields the unique solution the given system. If r<n, then n-r unknowns in the
reduce system can be chosen arbitrarily and the reduced system yields infinitely many
solutions of the given system.

1. : Solve the following system of linear equations by the Gauss elimination method
x1  x2  x3  4
2 x1  x2  x3  1
x1  x2  2 x3  2
1 2 1
Sol: For the given system, the coefficient matrix is A   2 1  1
1  1 2 
1 1 1 : 4 
And the augmented matrix is  A : B    2 1  1: 1 
1  1 2 : 2 
We reduce this matrix  A : B  to the upper triangular form by using elementary operations
Using the row operation R2  R2  2R1 and R3  R3  R1 ,We get
1 1 1 : 4 
 A : B   0  1  3:  7 
0  2 1 : 2 
Now, Using the row operation R 3  R 3  2 R 2 in this, we get
1 1 1 : 4 
 A : B   0  1  3:  7 
0 0 7 : 12 
We note that A is now reduced to the upper triangular form. The linear equations which
correspond to this reduced form of  A : B  are

DEPT OF MATHS, SJBIT Page 137


ENGINEERING MATHEMATICS-I 15MAT11

x1  x2  x3  4
 x2  3x3  7
7 x3  12
From equation (iii), wew find that x3  12 / 7.
36 13
x2  7  3x3  7  
7 7
Substituting for x3and x2 found above in (i), we get
13 12 3
x1  4  x2  x3  4   = .
7 7 7
Thus, x1 =3/7, x2 =13/7, x3 =12/7 constitute the solutionof the given system.

2. Solve the following system of equations by Gauss‟s elimination method:


4 x1  x2  x3  4
x1  4 x2  2 x3  4
3x1  2 x2  4 x3  6
4 1 1 : 4 
Sol: The augmented matrix is  A : B   1 4  2: 4 
3 2  4 : 6 
4 1 1 :4
 A : B   0 15 / 4  9 / 4 : 3 

0  10 2 : 6 
Using R 2  R 2  (1/ 4)R1 , R 3  R 3  3R 2
4 1 1 : 4 
 A : B   0 5  3 : 4 
0  5 1 : 3
Using R 2  (4 / 3)R 2 , R 3  (1/ 2)R 3
4 1 1 : 4 
 A : B   0 5  3 : 4
0 0  2 :1 
Using R 3  R 3  R 2 ,
We note that A is now reduced to the upper triangular form. The equations that
correspond to (i) are

DEPT OF MATHS, SJBIT Page 138


ENGINEERING MATHEMATICS-I 15MAT11

4 x1  x2  x3  4
5 x2  3x3  4
2 x3  1.
These yield .
1 1 1 1
x3   , x2  (4  3 x3 )  , x1  (4  x2  x3 )  1.
2 5 2 4
Thus, x1 =1, x2 =1/2, x3 =-1/2 constitute the solution of the given system.

3. Solve the following system of equations by Gauss‟s elimination method:


x  2y  2y 1
2x  y  z  2
3x  2 y  2 z  3
x z 0
1 2 2 :1 
 2 1 1: 2 
Sol: The augmented matrix is  A : B    
 3 2 2 : 3
 
0 1 1 : 0 
1 2 2 :1 
0  3  3: 0 
 A : B   0  4  4: 0 
 
0 1 1 : 0 
Using R 2  R 2  2R1 , R 3  R 3  3R1
1 2 2 :1
0 1 1: 0 
 A : B   0 1 1: 0 
 
0 1 1 : 0
Using R 2  (1/ 3)R 2 , R 3  (1/ 4)R 3
1 2 2 :1 
0 1 1: 0 
 A : B   0 0 0: 0 
 
0 0 0 : 0 
Using R 3  R 3  R 2 , R 4  R 4  R 2 ,

We note that A is now reduced to the echelon form. The system correspond to (i) is
x  2 y  2z  1
yz 0

DEPT OF MATHS, SJBIT Page 139


ENGINEERING MATHEMATICS-I 15MAT11

These are two equations for three unknowns. Therefore, we can choose one of the un-
known arbitrarily. Taking z=k, we get y=-k and x=1
Thus x=1,y=-k, z=k, Where k is arbitrary, is a solution of the given system.

5) Solve the following system of equations by Gauss‟s elimination method:


5 x1  x2  x3  x4  4
x1  7 x2  x3  x4  12
x1  x2  6 x3  x4  5
x1  x2  x3  x4  6
Sol : Consider augmented matrix  A : B  by R1  R4
 1 1 1 4 : 6 
 1 7 1 1 : 12 
 A : B    1 1 6 1 : 5 
 
 5 1 1 1 : 4
R2  R2  R1 , R3  R3  R1 , R4  R4  5R1
 1 1 1 4 : 6   1 1 1 : 6 
4
 0 6 0 3 : 18   0 2 0 1 : 6 
 A : B    0 0 5 3 : 1 

0 0 5 3 : 1 
   
 0 4 4 19 : 34   0 4 4 19 : 34 
R4  R4  2 R2
 1 1 1 4 : 6 
 0 2 0 1 : 6 
 A : B    0 0 5 3 : 1
 
 0 0 4 21 : 46 
R4  5 R4  4 R3
 1 1 1 4 : 6 
 0 2 0 1 : 6 
 A : B    0 0 5 3 : 1 
 
 0 0 0 117 : 234 
Hence we have x1  x2  x3  4 x4  6
2 x2  x4  6
5 x3  3 x4  1
117 x4  234
 x4  2, x3  1, x2  2 and x1  1is the reqd . so ln .

DEPT OF MATHS, SJBIT Page 140


ENGINEERING MATHEMATICS-I 15MAT11

Gauss -Jordan method:


This method can be regarded as the modification of Gauss – elimination method.
This method aims in reducing the coefficient matrix A to a diagonal matrix.
1) Applying Gauss Jordan method solve 2 x  3 y  z  5 , 4x  4 y  3z  3 , 2x  3 y  2z  2
 2 3 1 : 5 

Soln:  A : B   4 4 3 : 3 

 2 3 2 : 2 
R2  R2  2 R1 , R3  R3  R1
 2 3 1 : 5   2 3 1 : 5 
 A : B   0 2 1: 7   0 2
 1: 7 
 0 6 3 : 3 0 2 1 : 1
R1  2 R1  3R2 , R3  R3  R2
 4 0 5 : 11  4 0 5 : 11
 A : B   0 2 1: 7   0 2
 1: 7 
 0 0 2 : 6   0 0 1 : 3 
R1  R1  5R3 , R2  R2  R3
4 0 0 : 4
 A : B   0 2 0 : 4
 0 0 2 : 6 
Hence 4 x  4, 2 y  4, 2 z  6
 x  1, y  2, z  3

2.

DEPT OF MATHS, SJBIT Page 141


ENGINEERING MATHEMATICS-I 15MAT11

DEPT OF MATHS, SJBIT Page 142


ENGINEERING MATHEMATICS-I 15MAT11

DEPT OF MATHS, SJBIT Page 143


ENGINEERING MATHEMATICS-I 15MAT11

Problem 1

DEPT OF MATHS, SJBIT Page 144


ENGINEERING MATHEMATICS-I 15MAT11

Problem 2

DEPT OF MATHS, SJBIT Page 145


ENGINEERING MATHEMATICS-I 15MAT11

Problem 3

Soln:

DEPT OF MATHS, SJBIT Page 146


ENGINEERING MATHEMATICS-I 15MAT11

DEPT OF MATHS, SJBIT Page 147


ENGINEERING MATHEMATICS-I 15MAT11

LINEAR TRANSFORMATION:

A Linear transformation in two dimensions is given by


y1  a1 x1  a2 x2
y2  b1 x1  b2 x2
This can be represented in the matrix form as
 y1   a1 a2   x1 
     ) Y  AX
 y2  b1 b2   x2 
Similarly a linear transformation in 3 dimensions along with its matrix form is as,
y1  a1 x1  a2 x2  a3 x3
y2  b1 x1  b2 x2  b3 x3
y3  c1 x1  c2 x2  c3 x3
A is called transformation matrix
If A  0 Then y=AX is called non-Singular transformation or regular transformation.
If A  0 Then y=AX is called Singular transformation
X= A1 y is called the inverse transformation.
Let z=By =B(AX)=(BA)X=CX Where C=BA Z=CX is called a composite linear
transformation .
1. Show that the transformation
y1  2 x1  x2  x3 ; y2  x1  x2  2 x3 y3  x1  2x3 is regular. Write down the
inverse transformation.
Sol: The given transformation may be written as
Y=AX
 y1   2 1 1   x1 
     
 y2   1 1 2   x2 
 y  1 0  2   x 
 3    3
2 1 1 
A  1 1 2   2(2)  1( 2  2)  1(1)  4  4  1   1  0
1 0  2 
A  0  A is a non-singular matrix
The transformation is regular
The inverse transformation is X= A1Y

DEPT OF MATHS, SJBIT Page 148


ENGINEERING MATHEMATICS-I 15MAT11

 2  2 1 
adjA 
1
A    4 5 3
A
 1  1  1 
X  A1Y
 x1   2  2  1   y1 
 x    4 5 3  
 2    y2 
 x3   1  1  1   
 y3 
x1  2 y1  2 y2  y3
x2  4 y1  5 y2  3 y3 is the inverse transformation.
x3  y1  y2  y3

2. Prove that the following matrix is orthogonal


 2 1 2 
 3 3 3 

A 2 2 1 
 3 3 3 
 1 2 2 
 3 3 3 
 2 1 2   2 2 1 
 3 3 3  3 3 3  1 0 0 
Sol:Consider AA  I   2 2 1  1 2 2   0 1 0 
 3 3 3  3 3 3  
 1 2 2  2 1 2  
 0 0 1 
 3 3 3   3 3 3 

1 2 a 
3. If A=1/3  2 1 b  is orthogonal. Find a,b,c & A1 A is orthogonal AA  I
 2  2 c 
1 2 a  1 2 2  1 0 0 
1   1 
Sol :  2 1 b   2 1  2   0 1 0 

3 3
 2  2 c   a b c  0 0 1 
1  4  a 2 2  2  ab 2  4  ac  9 0 0 
 
 2  2  ab 4  1  b
2
4  2  bc   0 9 0 
 2  4  ac 4  2  bc 4  4  c 2  0 0 9 

5  a2  9 5  b2  9 8  c2  9
a2  4 b2  4 c2  1
a2 b2 c 1

DEPT OF MATHS, SJBIT Page 149


ENGINEERING MATHEMATICS-I 15MAT11

1 2 2 
AA  I  A  A   2 1  2 
1
1

3
 2 2 1 

4. Find the inverse transformation of the following linear transformation


y1  x1  2 x2  5 x3
y2  2 x1  4 x2  11x3
y3   x1  2 x3
Sol: Y=AX
 x1   19  9 2  y1 
 
A Y   x2    4
1
2  1  y2 
 x3   2 0   
1  y3 
x1  19 y1  9 y2  2 y3
x2  4 y1  2 y2  y3 is the inverse transformation
x3  2 y1  y2
5. Represents each of the transformation y1  z1  2 z2 & x2   y1  4 y2 , y2  3z
by the use of matrix & find the composite transformation which express
x1 , x2 in terms of z1 , z2
Sol : x1  3 y1  2 y2
x2   y1  4 y2
x   3 2   y1 
 x  AY   1   
 x2   1 4   y2 
y1  z1  2 z2
 y1  1 2   z1 
y2  3 z1  y  BZ       
 y2   3 0   z 2 
X  AY  A( BZ )  ABZ   AB  Z
 3 2  1 2  9 6 
 AB    1 
4  3 0  11  2 

 x1  9 6   z1 
 x   11  2   z 
 2   2
x1  9 z1  6 z2
x2  11z1  2 z2 is the required composite transformation

6.` Given the linear transformation

DEPT OF MATHS, SJBIT Page 150


ENGINEERING MATHEMATICS-I 15MAT11

y1  5 x1  3 x2  3 x3
y2  3 x1  2 x2  2 x3
y3  2 x1  x2  2 x3

Sol : z1  4 x1  2 x3
z2  x2  4 x3
z3  5 y3
Express y1, y2, y3
int erms of z1 , z2 , z3
Given : Y=AX
1 0 1 
 4 10 
Z  BX  X  B 1Z B 1 0 1 4 
 5 
0 0 1 
 5 
Y  ( AB 1 ) Z
5 3 23 
 y1   4 10   Z1 
  3 23   z 
 y2    4 1 10   2 
y     z3 
 3 1 1 1
 2 

Eigen values and Eigen vectors of a square matrix:

Definition: Let A be a given square matrix of order n. suppose I a non-zero Column


vector X of order n and real or complex no.  Such that AX=  x
Then X is called an Eigen vector of A.
 is called the corresponding Eigen value of A.
Working Rule:
1. Given square matrix A write down the characteristic equation A   I  0
2. Solve the characteristic equation for Eigen values 1 , 2 , 3 ,.......
3. To find Eigen vector, write down the matrix equation as
 x1 
 A   I  X  0 where X   x2 
 xn 
4. We set   1 in the matrix equation & solve it for Eigen vector x1 . Similarly we
obtain Eigen vector x2 , x3 ..... for corresponding Eigen value 2 , 3 .....

DEPT OF MATHS, SJBIT Page 151


ENGINEERING MATHEMATICS-I 15MAT11

1. Find the Eigen values & corresponding eigen vector of the following matrix
 3 8 
A 
 2 7 
3   8
Sol : A   I   (3   )(7   )  16
2 7
  2  4  5
 (  5) (  1)  0
The roots of this equation are 1 =5 & 2 =-1. These are the two Eigen value of the given
matrix A. Let X=  x, y  , then the matrix equation ( A   I ) X  0
T

 3   8   x  0
 
 2 7     y  0
  1  5
 8 8   x  0 
 2 2   y   0 
    
-8x+8y=0, -2x+2y=0. Both of these reduces to some equation x-y=0 =) x=y. If we choose
a 
x=a, then y=a These, when   1  5, x1    is the solution of (1)
a 
0  2  1 equation (1) becomes
-2 8  x  0 
-2 8  y   0 
    
For 2 x  8 y  0, x  4 y  0.Hence if we choose y=b, then x=4b
 Ab 
Thus when   2  1, x2    is the soln of (1)
b 

2. Find the Eigen values & the Eigen vectors of the matrix
2 0 1 
A  0 2 0 
1 0 2 
Sol: For the given matrix, the characteristic polynomial is
2   0 1 

A   I  0 2   0 
1 0 2   
 (2  x)3  (2   )
 (  3)(2   )(  1)

DEPT OF MATHS, SJBIT Page 152


ENGINEERING MATHEMATICS-I 15MAT11

The characteristic equation of the given matrix is


(2   )(  3)( 1)  0
The roots are 1  1 2  2 & 3  3 these are the Eigen value of the given matrix.
 x, y , z   X , then the matrix equation (A- I)x=0
T

 2- 0 1   x  0
0 2   0   y    0   (1)
    
1 0 2     z   0 
For   1  1
1 0 1  x   0 
0 1 0   y   0
    
1 0 1  z   0 
x z 0 & y 0
If we choose x=a, then z=-a; x1   a1 01  a 
T

x1  1 0  1
T
If a  1
0 0 1  x  0
For   2  2 0 0 0   y   0
   
1 0 0   z   0 
x  0, z  0. we take y=b; x 2  0 b 0 
T

x 2   0 2 0
T
If b  2
Let  1 0 1   x  0 
For   3  3 0  1 0   y   0 
1 0  1  z   0 
 x  z  0, y  0. we take x=c; then z=c x 3   c o c 
T

x 3  3 0 3
T
If c  3

1 1 3
3. Find the matrix P which reduces the matrix A  1 5 1 to diagonal form
3 1 1
Hence find A4
1- 1 3 

A   I  1 5   1   0
3 1 1   
1  2 2  3 3  6

DEPT OF MATHS, SJBIT Page 153


ENGINEERING MATHEMATICS-I 15MAT11

1-
1 3 
A   I  15   1   0
31 1   
1  2 2  3 3  6
 3 1 3  x   0 
Case(1): 1 7 1   y   0 
3 1 3   z  0 
3x  y  3z  0
x  7y  z  0
x y z
 
1 3 3 3 3 1
7 1 1 1 1 7
x y z
 
20 0 20
Caseii ): 2  3
 2 1 3   x   0 
1 2 1   y    0 
    
3 1  2   z  0 
2 x  y  3 z  0
x  2y  z  0
3x  y  2 z  0
x y z
 
2 1 1 1 1 2
1 2 3 2 3 1
x y z
 
5 5 5
Caseiii ): 3  6
 5 1 3   x   0 
1  1 1   y    0 
    
3 1  5  z  0 
5 x  y  3 z  0
x yz 0
3x  y  5 z  0
x y z
 
1 1 1 1 1  1
1 5 3 5 3 1
DEPT OF  y z SJBIT
x MATHS, Page 154
 
4 8 4
ENGINEERING MATHEMATICS-I 15MAT11

x y z
 
4 8 4
 x1 x2 x3   1 1 1 
p   x1 x2 x3    y1 y2 y3   0  1 2 
 z1 z2 z3  1 1 1 
 1 1
 2 0 2 
 
1
p   1

1 1
 3 3 3
 
 1 1 1
 6 3 6 
 2 0 0 
p AP  0
1
3 0   D
0 0 6 
 1 1
 2 0 2 
 1 1 1  16 0 0  
A  PD p  0  1 2 
4 4 1 0
 81 0   1 1 1
 3 3 3
1 1 1  0 0 1296   
 1 1 1
 6 3 6 
 251 405 235 
  405 891 405 
 235 405 251 

11 4 7
5) Diagonalizable the matrix A  7 2  5  and find A5
10 4  6 
(A  I)  0
11    4  7 
7  2    5   0

10  4  6   
 3  3 3  2  0
  2  3  2   0
x  0  1 2

DEPT OF MATHS, SJBIT Page 155


ENGINEERING MATHEMATICS-I 15MAT11

Casei :   0
11 4 7   x  0
7 2  5   y   0
    
10 4  6   z  0 
11x  4 y  7 z  0
7 x  2 y  5z  0
10 x  4 y  6 z  0
x y z
 
2  5 7 5 7 2
4  6 10  6 10  4
x y z
 
8 8 8

Caseii :   1
10 4 7   x  0
7 3  5   y   0 
    
10 4  7   z  0 
10 x  4 y  7 z  0
7 x  3 y  5z  0
10 x  4 y  7 z  0
x y z
 
4  7 10  7 10  4
3  5 7  5 7  3
x y z
 
1 1 2
Caseiii :   2
9  4  7   x  0
 7  4  5  y   0
     
10  4  8  z  0 
9x  4 y  7z  0
7 x  4 y  5z  0
10 x  4 y  8 z  0

DEPT OF MATHS, SJBIT Page 156


ENGINEERING MATHEMATICS-I 15MAT11

x y z
 
4  5 7 5 7 4
4  8 10  8 10  4
x y z
 
12 6 12
1 1 2
p   x1 x2 x3   0  1 1 
1 2 2 
 4 2 3 
p   1 0
1
1 
3  1  2 
0 0 0
p 1 AP  0 1 0   D
0 0 2 
191  64  127 
A5  PD 5 p 1  97  32  65 
190  64  126 

Quadratic forms:
A homogeneous expression of the second degree in any number of variables is
called a quadratic form (Q.F).
In general for two variables x1 , x2 i.e, n  1, 2 a11 x12  2a12 x1 x2  a22 x22
is called QF in
two variables.

 coeff of x1 x2 
2 1
coeff of x1 2
The matrix A of the above Q.F is A   
 1  coeff of x x  coeff of x 2 
 2 
1 2 1

 1 
1 2 
Eg : x 2  y 2  xy A 
1 1 
 2 
2 3
2) 2 x 2  3 y 2  6 xy A
3 3 
5 6
3) 5 x12  7 x22  12 x1 x2 A
6 7 

DEPT OF MATHS, SJBIT Page 157


ENGINEERING MATHEMATICS-I 15MAT11

Similarly Q.F in 3 variables is


a11 x12  a22 x22  a33 x32  2a12 x1 x2  2a13 x1 x2  2a23 x1 x2
 1 1 
coeff of x1
2
coeff  x1 x2  coeff  x1 x3  
2 2
 
 1 Coeff of  x x  coeff of x 2 1
coeff  x2 x3  
A  2 1 2 1
2 
1 
 Coeff of  x1 x2  1 coeff  x2 x3  coeff of x32 
2 2 
 
 
 1 
 1 2
2
 
A 
1
Examples :1) QF : x 2  y 2  z 2  xy  2 yz  4 zx 1
 2 
 2 1 1 

 
5 2 0 
2) 5 x  2 yz  6 y  9 z  4 xy
2 2 2
A   2 6 1 
0 1 9 

 1 1
 0
2 2
 
3) xy  yz  zx A  
1 1
0
 2 2
 1 1 
 0 
 2 2 

Canonical Form (sum of squares):


Q  1 y12  2 y22  .......  n yn2 where 1 , 2 ,.......n are Eigen values called canonical form.
Rank Index & Signature of canonical form

The number of non-zero terms present in a canonical form of Q is called rank of Q it, (r)
Ex :2 y12  y22  y32  r  3 y12  y32  r  2

1. The number of the terms present in a canonical form is called index of Q. (p)
Ex :2 y12  3 y22  5 y32 p  2

2. The difference between the negative terms in a canonical form is called signature
of Q (s)
DEPT OF MATHS, SJBIT Page 158
ENGINEERING MATHEMATICS-I 15MAT11

Ex : y12  3 y22  y32 s  2 1  1

Nature of Quadratic Form:


r=rank, p=index n=number of variables
Condition Meaning Nature of Q.F Eg;
r=n,p=n All n Co efficient +ve definite 2 y12  y22  8 y32 +
are positive
r=n,p=0 All n Co efficient -ve definite  y12  y22  6 y32
are
-ve
r=p,p<n for r=2=p At least one of the +ve Semi y22  5 y32
2<3 Co-efficient Zero definite
& all other Co-
efficientp are +ve
r<n,p=0 At least one of the -ve Semi definite  y22  10 y32
Co-efficient Zero
& all other Co-
efficient are -ve
Note: Q.F is indefinite if some of the Co-efficient are +ve and some are -ve
Eg: y12  y22  3 y32

Working rule to reduce Q.F to Canonical (sum of squares) form by


orthogonal transformation.

1. Write down the matrix A to Q.F


2. Find the Eigen values & the corresponding eigen vectors of matrixA.
3. Normalise the Eigen vector x1 , x2 , x3
x1
i.e, x11 
x1
a 
If x1  b   x11  a 2  b 2  c 2
c 
x x
111iy x12  2 x31  3
x2 x3
4. Write down the associated orthogonal model matrix Q   x11 x12 x31 
5. Since pp1  I  p 1  p1
Then p 1 AP  p1 AP  diagonal matrix  1 , 2 , 3 

DEPT OF MATHS, SJBIT Page 159


ENGINEERING MATHEMATICS-I 15MAT11

6. The associated canonical form is 1 y12  2 y22  3 y32


 x1   y1 
   
7. x=py where X   x2  & Y   y2 
 x3   y3 

will give us the orthogonal linear transformation.

1) Obtain the canonical form of the quadratic form


Sol : 2 x 2  2 y 2  2 z 2  2 xy  2 yz  2 zx
 2  1  1
A   1 2  1
 1  1 2 
(A  I )  0
2    1  1 
  1 2    1   0
 1  1 2   
 
(2   ) (2   ) 2  1  1(2   )  1  11  (2   )  0
(2   ) 4    4  1  2    1  3     0
2

2    4  3  3    3    0
2

2 2  8  6   3  4 2  3  3    3    0

 3  6 2  9  0   3  6 2  9  0
 ( 2  6  9)  0  (  3)3  0
  0,3,3 i.e, 1  0, 2  0, 3  0 are roots and are the Eigen values of A.

2) The canonical form of the given Q.P that we get by an orthogonal transformation
is 1 y12  2 y22  3 y32  3 y22  3 y32
Sol: Since one Co- efficient in this canonical form is zero & the other two are +ve,
the Q.F is +ve Semi-definite
Rank, r=2 Index, p=2 & Signature, s=2.

3) Reduce the Q.F x12  3x22  3x32  2 x2 x3 to the canonical form by an orthogonal
transformation
1 0 0 
Sol : A  0 3  1
0  1 3

DEPT OF MATHS, SJBIT Page 160


ENGINEERING MATHEMATICS-I 15MAT11

(A  I)  0
1  0 0
0 3 1 0
0 1 3
 
(1   ) (3   ) 2  1  0  0
(1   ) 9    6  1  0
2

(1   )    6  8   0
2

 2  6  8   3  6 2  8  0
 3  7 2  14  8  0   3  7 2  14  8  0
(  1) (  2) (  4)  0
The eigen values of A are 1  1 2  2 3  4
Casei : For 1  1
0 0 0   x1  0 
0 2  1  x   0 
   2  
0  1 2   x3  0 
x y z
 
2 1 0 1 0 2
1 2 0 2 0 1
x y z
 
3 0 0
3  1 
x1  0   0 
0  0 

11x111  12  0  0  1
100  1 0
T
x
 0
T
x  1 
1
1
x1 1
2  2
 1 0 0   x1  0 
0 1  1   x   0 
   2  
0  1 1   x3  0 
x y z
 
1 1 1 0 1 0
1 1 0 1 0 1

DEPT OF MATHS, SJBIT Page 161


ENGINEERING MATHEMATICS-I 15MAT11

x y z
 
0 1 1
3 
x2  1 
 1
11x211  0  1  1  2
T
x 1  1 1 
x2  2    0 1  1  0
T
 
x2 2  2 2

3  4
 3 0 0   x1  0 
0  1  1   x   0 
   2  
0  1  1  x3  0 
x y z
 
1  1 3 0 3 0
1  1 0 1 0 1
x y z
 
0 3 3
0  0 
x1   3   1
3  1 
11x311  0  1  1  2
0  1 1  1 1 1 
T T
x
x  3 
1
3  
x3 2  2 2
The orthogonal model matrix for A is
 
1 0 0 
 
 1 1 
Q   x11 x12 x31   0
 2 2 
 1 1 
0  2 2 
 
X=Q.F is the orthogonal transformation that reduces the given Q.F to the canonical form.
The canonical form is
1 y12  2 y22  3 y32  y12  2 y22  4 y32

DEPT OF MATHS, SJBIT Page 162


ENGINEERING MATHEMATICS-I 15MAT11

Rayleigh‟s power method is an iterative method to determine the numerically largest


eigen value and the corresponding eigen vector of a square matrix.

Working procedure:
 Suppose A is the given square matrix, we assume initially an eigen vector X 0 in a
simple for like [1,0,0] or [0,1,0] or [0,0,1] or [1,1,1] and find the matrix
product AX 0 which will also be a column matrix.
1
 We take out the largest element as the common factor to obtain AX 0   1 X .
1 1  2
 We then find AX and again put in the form AX    2 X by normalization.
 The iterative process is continued till two consecutive iterative values of λ and X
are same upto a desired degree of accuracy.
 The values so obtained are respectively the largest eigen value and the
corresponding eigen vector of the given square matrix A.

Problems:
1) Using the Power method find the largest eigen value and the corresponding eigen
vector starting with the given initial vector.
2 0 1 
0 2 0 given 1 0 0T
 
1 0 2
 2 0 1  1   2  1 
Solution : AX  0
 0 2 0  0    0   2  0    1 X 1
     
1 0 2  0  1  0.5
 2 0 1   1   2.5 1 
AX 1   0 2 0   0    0   2.5  0     2 X  2
     
1 0 2 0.5  2  0.8
 2 0 1   1   2.8  1 
AX  2
 0 2 0   0    0   2.8  0    3 X  3
     
1 0 2  0.8  2.6  0.93

DEPT OF MATHS, SJBIT Page 163


ENGINEERING MATHEMATICS-I 15MAT11

2 0 1   1   2.93  1 
AX   0
 3
2 0   0    0   2.93  0     4 X  4 
    
1 0 2  0.93  2.86  0.98
2 0 1   1   2.98  1 
 4 
AX   0 2 0   0    0   2.98  0     5 X  5
    
1 0 2  0.98  2.96  0.99 
2 0 1   1   2.99   1 
 5 
AX   0 2 0   0    0   2.99  0     6  X  6 
     
1 0 2  0.99   2.98  0.997 
 2 0 1   1   2.997   1 
AX   0 2 0   0    0   2.997  0     7  X  7 
 6

1 0 2  0.997   2.994  0.999 


Thus the largest eigen value is approximately 3 and the corresponding eigen vector is
[1 ,0, 1]‟

2) Using the Power method find the largest eigen value and the
corresponding eigen vector starting with the given initial vector.
 4 1  1
 2 3  1 given 1 0.8  0.8
T
 
 2 1 5 
4 1 1  1   5.6   1 
 0 
Solution : AX   2 3 1  0.8    5.2   5.6  0.93    1 X 1
     
 2 1 5   0.8  5.2  0.93
4 1 1  1   5.86   1 
AX   2
1
3 1  0.93    5.72   5.86  0.98     2 X  2
     
 2 1 5   0.93  5.72   0.98
4 1 1  1   5.96   1 
 2 
AX   2 3 1  0.98    5.92   5.96  0.99     3 X 3
     
 2 1 5   0.98  5.92  0.99
4 1 1  1   5.98   1 
AX   2
 3
3 1  0.99    5.96   5.98  0.997     4 X  4
     
 2 1 5   0.99   5.96  0.997 
4 1 1  1   5.994   1 
 4 
AX   2 3 1  0.997    5.988   5.994  0.999     5 X  5
     
 2 1 5   0.997   5.988  0.999 

DEPT OF MATHS, SJBIT Page 164


ENGINEERING MATHEMATICS-I 15MAT11

Thus after five iterations the numerically largest eigen value is 5.994 and corresponding
eigen vector is [1, 0.999, -0,999]‟

6) Using Rayleigh‟s power method to find the largest Eigen value and the corresponding
Eigen vector of the matrix.
(Dec 2012)
 6 2 2 
Sol: A   2 3  1 X (0)  1,0,0
 2  1 3 
6  2 2  1  6   1 
AX (0 )
  2 3  1 0   2  6 0.333
    
 2  1 3  0  2   0.3333 
6  2 2  1   7.3332   1 
AX (1 )   2 3  1  0.333    3.3332  7.3332 0.4545
     
 2  1 3   0.3333   3.3332   0.4545 
 6  2 2  1   7.818   1 
AX   2 3  1  0.4545    3.818  7.818 0.488
(2 )      
 2  1 3   0.4545   3.818   0.488 
 6  2 2  1   7.952   1 
AX   2 3  1  0.488    3.952  7.952 0.4969
(3 )       
 2  1 3   0.488   3.952   0.4969 
The largest Eigen value is   7.952 and the corresponding Eigen vector is
1  0.4969 0.4969

DEPT OF MATHS, SJBIT Page 165

Вам также может понравиться